pathophysiology midterm

Lakukan tugas rumah & ujian kamu dengan baik sekarang menggunakan Quizwiz!

A nurse is developing a program to help clients reduce the impact of chronic stress on their health. Which clients should the nurse prioritize as being able to most benefit from the program? select all that apply

-A 22 yr old with an eating disorder -A 30 yr old with substance abuse -A 45 yr old with inflammatory bowel disease

A widow who lost her husband a few weeks ago is having trouble with insomnia. When visiting with her health care provider, the provider suggests a prescription to help her regain a normal circadian pattern. This is based on the fact that interruption of sleep-wake cycles can cause which problems? Select all that apply

-An increased risk in accidents when sleep deprived, similar to those under the influence of alcohol -Alterations in immune function that can result in an infection

The nurse is caring for a client who has a tumor that results in excess levels of catecholamines being released. Which assessments will the nurse prioritize in this client's plan of care? Select all that apply

-Blood glucose -Blood pressure -Heart rate and rhythm

The ICU nurse is concerned with her client's arterial blood gas results-especially the pH 7.30; and PCO2 49 mmHg. The nurse interprets these ABG results and assesses her client for which clinical manifestations of respiratory acidosis? Select all that apply

-Muscle twitching -Headache with complaints of blurred vision

A nurse is caring for a client with posttraumatic stress distress disorder. Which behaviors would the nurse expect the client to manfest? Select all that apply

-Reluctance to participate in group discussions -Difficulty concentrating on tasks -Sleep disturbance with vivid nightmares

A client is brought to the emergency dept for an overdose of aspirin. The nurse caring for this client should anticipate which clinical manifestations? Select all that apply.

-Respiratory rate of 40 -ABG report: pH 7.50, PCO2 31 mmHg, HCO3 level 19 mmol/L

During a period of extreme excess fluid volume, a renal dialysis client may be administered which type of IV solution to shrink the swollen cells by pulling water out of the cell?

3% sodium chloride

Viruses are killing some of his B cells and becoming incorporated into the genome of others.

A 14-year-old boy has been diagnosed with infectious mononucleosis. Which pathophysiologic phenomenon is most responsible for his symptoms? The Epstein-Barr virus (EBV) is lysing many of the boy's neutrophils. The virus responsible for mononucleosis inhibits the maturation of myeloblasts into promyelocytes. EBV inhibits the maturation of white cells within his peripheral lymph nodes. Viruses are killing some of his B cells and becoming incorporated into the genome of others.

Chest auscultation reveals crackles in her lower lung fields bilaterally.

A 16-year-old female has been brought to her primary care physician by her mother due to the girl's persistent sore throat and malaise. Which fact revealed in the girl's history and examination would lead the physician to rule out infectious mononucleosis? The girl has a temperature of 38.1°C (100.6°F) and has enlarged lymph nodes. Chest auscultation reveals crackles in her lower lung fields bilaterally. Her liver and spleen are both enlarged. Blood work reveals an increased white blood cell count.

"Your prognosis will depend on whether we can surgically resect your tumor."

A 20 year-old has been diagnosed with an astrocytic brain tumor located in the brainstem. Which statement by the oncologist treating the client is most accurate? Our treatment plan will depend on whether your tumor is (malignant or benign." This is likely a result of a combination of heredity and lifestyle. "The major risk that you face is metastases to your lungs, liver, or bones." "Your prognosis will depend on whether we can surgically resect your tumor."

A decline in functioning acetylcholine receptors; treatment with corticosteroids and intravenous immunoglobulins

A 22-year-old female college student is shocked to receive a diagnosis of myasthenia gravis. What are the etiology and most likely treatment for her health problem? Autoimmune destruction of skeletal muscle cells, treatment with intensive physical therapy and anabolic steroids. Cerebellar lesions; surgical and immunosuppressive treatment Excess acetylcholinesterase production; treatment with thymectomy. A decline in functioning acetylcholine receptors; treatment with corticosteroids and intravenous immunoglobulins

Tonic-clinic seizure

A 26 year-old female is resting after a 1-minute episode during which she lost consciousness while her muscles contracted and extremities extended. This was followed by rhythmic contraction and relaxation of her extremities. On regaining consciousness, she found herself to have been incontinent of urine. What has the woman most likely experienced? A myoclonic seizure An absence seizure • A tonic-clonic seizure A complex partial seizure

Heart failure resulting from childhood chemotherapy

A 26-year-old man who survived childhood acute lymphocytic leukemia (ALL), one of the most common childhood cancers, now complains of weakness, fatigue, and shortness of breath. His treatment for ALL likely included anthracyclines. What is the most likely cause of his symptoms? •CNS problems resulting from childhood chemotherapy •Hormonal dysfunction resulting from childhood chemotherapy •Recurrence of ALL • Heart failure resulting from childhood chemotherapy

Proliferation of immature neutrophils Increased segmented neutrophil

A 29-year-old construction worker got a sliver under his fingernail 4 days ago. The affected finger is now reddened, painful, swollen, and warm to the touch. Which of the following hematologic processes is most likely occurring in the bone marrow in response to the infection? High circulatory levels of myeloblasts Phagocytosis by myelocytes Proliferation of immature neutrophils Increased segmented neutrophil production

Multiple myeloma

A 30 year-old male's blood work and biopsies indicate that he has proliferating osteoclasts that are producing large amounts of IgG. What is the man's most likely diagnosis? Hodgkin lymphoma Multiple myeloma Acute myelogenous leukemia Acute lymphocytic leukemia

"So much clotting takes place that there are no available clotting components left, and bleeding ensues."

A 30-year-old woman who has given birth 12 hours prior is displaying signs and symptoms of disseminated intravascular coagulation (DIC). The client's husband is confused as to why a disease of coagulation can result in bleeding. Which statement by the nurse best characterizes DIC? *Massive clotting causes irritation, friction, and bleeding in the small blood vessels. "These same hormones and bacteria that cause clotting also cause bleeding "So much clotting takes place that there are no available clotting components left, and bleeding ensues." Excessive activation of clotting causes an overload of vital organs, resulting in bleeding.

Accelerated chronic myelogenous leukemia

A 32-year-old woman presents at her health clinic complaining of weakness, feeling of abdominal fullness, 6-month history of fatigue and night sweats. She added a multivitamin with iron and some extra meat and leafy greens to her diet but has not experienced an increase in energy. Upon assessment, her spleen was noted to be enlarged. Which diagnosis is most likely associated with her manifestations? Chronic lymphocytic leukemia Infectious mononucleosis Stage A Hodgkin disease Accelerated chronic myelogenous leukemia

"We need to ensure your birth control pills don't contain estrogen."

A 36-year old woman with a diagnosis of antiphospholipid syndrome is receiving a scheduled checkup from her nurse practitioner. Which teaching point would the nurse most likely prioritize? "Good nutrition and blood sugar control are important in your case!! You'll need to avoid taking nonsteroidal anti-inflammatory drugs when you have menstrual cramps." "It's important for you to do regular physical activity and maintain a healthy body weight." "We need to ensure your birth control pills don't contain estrogen."

The lymph nodes are usually affected, and often the spleen and bone marrow."

A 40-year-old male client is shocked to receive a diagnosis of mature B-cell lymphoma, and is doing research on his diagnosis on the Internet. Which statement that he reads on various websites is most reliable? "Doctors are able to diagnose mature B-cell lymphoma by the presence of Reed-Sternberg cells." "The lymph nodes are usually affected, and often the spleen and bone marrow." "Unlike many other lymphomas, mature B-cell lymphoma is often self-limiting and treatment is focused on symptoms." "Like most forms of Hodgkin lymphoma, mature B-cell lymphoma often requires radiation treatment."

A tumor is undetectable until it has doubled 30 times and contains at least 1 billion cells.

A 41-year-old female with a family history has had a baseline mammogram. She states that she performs monthly self breast exams but really has a hard time evaluating her lumps since she has numerous cysts. At her annual mammogram, the technician views a suspicious area and refers her to the radiologist. She asks the nurse in the office, "How can a lump appear so quickly?" The nurse's response is based on which principle? -A tumor is undetectable until it has doubled 30 times and contains at least 1 billion cells. -If the breast has a lot of cysts, then the fluid within those sacs makes it hard to feel the hard lumps of a cancer. -Cancer cells are undifferentiated and come in various shapes and sizes -Many tumor cells never leave the M phase of the cell cycle

Vitamin K deficiency

A 44-year-old female client presents to the emergency department with abnormal bleeding and abdominal pain that is later attributed to gallbladder disease. Which diagnosis would the medical team be most justified in suspecting as a cause of the client's bleeding? Excess calcium Hemophilia B Vitamin K deficiency Idiopathic immune thrombocytopenic purpura (ITP)

Insufficient levels of neutrophils make him particularly susceptible to infections.

A 44-year-old hospitalized client with a diagnosis of end-stage acquired immunodeficiency syndrome (AIDS) has been placed on neutropenic precautions that limit his interaction with visitors, staff, and other clients. What is the underlying rationale for these precautions? His antibody-mediated immunity is compromised by his low production of neutrophils. Cyclic neutropenia limits his body's ability to fight various infections. Insufficient levels of neutrophils make him particularly susceptible to infections. Neutropenia limits the ability of his CD4+ helper cells to present antigens.

Assessment of swallowing ability and respiratory status.

A 47-year-old woman was diagnosed with amyotrophic lateral sclerosis 3 years ago and has experienced a progressive onset and severity of complications. She has been admitted to a palliative care unit due to her poor prognosis. What assessments and interventions should the clinical staff of the unit prioritize in their care? Regular pain assessment and administration of opioid analgesics as needed Assessment and documentation of cognitive changes, including confusion and restlessness Cardiac monitoring and administration of inotropic medications. • Assessment of swallowing ability and respiratory status.

posterior lilac crest

A 5-year-old needs to undergo a bone marrow biopsy. When educating the parents, which site should the nurse state is the most commonly used? sternum spinous. anterior iliac crest posterior iliac crest

Immunohistochemistry

A 51-year-old female has been found to have metastatic lesions in her lung but her oncologist is unsure of the primary tumor site. Which procedure is most likely to aid in this determination? •Microarray technology • Immunohistochemistry •Tumor markers •Tissue biopsy

To reduce the viscosity of his blood

A 53-year-old man presents with inability to concentrate, itching in his fingers and toes, elevated blood pressure, and unexplained weight loss. He is diagnosed with primary polycythemia. What will be the primary goal of his treatment? To increase the amount of oxygen distributed by his red blood cells To reduce the viscosity of his blood To reduce the mean size of his red cells To control his hypertension

The boy will pass the gene to all his future daughters, who will become carriers.

A 6-year-old boy with intellectual disability secondary to fragile X syndrome has been admitted to the hospital with a mitral valve prolapse. The health care worker caring for the child should have which concepts as part of her knowledge base regarding fragile X syndrome? -Genes of the boy's Y chromosome can be affected in addition to the X chromosome -The boy's mother had a 100% chance of transmitting the defective gene to her son. -The boy will pass the gene to all his future daughters, who will become carriers. -An affected mother who carries one normal and one mutant allele has a 75% chance of transmitting the gene to her daughters.

tests of kidney function

A 6-year-old girl with a diagnosis of Marfan syndrome is being assessed at a community health clinic. Which assessment would be the health care professional's lowest priority? Tests of kidney function A musculoskeletal assessment A test of the child's visual acuity Cardiovascular assessment

His onset of pain has been gradual and he has no prior history of lower back problems.

A 60-year-old male office worker presents to a clinic complaining of new-onset of lower back pain that has been worsening over the last 6 weeks. The physician knows that which component of his physical assessment and history is most indicative of a serious pathologic process (like aortic aneurysm or cancer)? His onset of pain has been gradual and he has no prior history of lower back problems. When supine, passive rising of his leg to 909 results in hamstring pain His pain is relieved by extended bed rest. He has needed regular nonsteroidal anti-inflammatory drugs to control the pain in recent weeks.

"Tumor markers alone aren't enough to confirm whether you have cancer or not, so they're not a very useful screening tool."

A 60-year-old man has presented to a clinic and is requesting screening for tumor markers after reading about them in a magazine. What can the clinician most accurately tell the man about the clinical use of tumor markers? -Tumor markers are a very useful screening tool, but they only exist for a very few types of cancer!" -"Tumor markers are an excellent screening tool, but it's only practical to test for those cancers that you're at risk of! -"Tests for the presence of tumor markers are limited by the fact that they are only accurate in the very early stages of cancer!! -"Tumor markers alone aren't enough to confirm whether you have cancer or not, so they're not a very useful screening tool."

The lymph nodes involved are located in a large number of locations in the lymphatic system.

A 60-year-old woman is suspected of having non-Hodgkin lymphoma (NHL). Which aspect of her condition would help to rule out Hodgkin lymphoma? The woman complains of recent debilitating fatigue. The presence of Reed-Sternberg cells has been confirmed. The lymph nodes involved are located in a large number of locations in the lymphatic system. Her neoplasm originates in secondary lymphoid structures.

Erythropoietin causes the erythrocyte colony-forming units to proliferate and mature.

A 62-year-old female with a diagnosis of acute and chronic renal failure secondary to diabetes mellitus is receiving her weekly injection of epoetin, a supplementary form of erythropoietin. Which statement best captures the necessity of this medication? Erythropoietin is needed in order for stem cells to proliferate into committed erythroid precursors. Erythropoietin is necessary for the accurate sensation of hypoxia that stimulates erythropoiesis. Erythropoietin facilitates the extrusion of the reticulocyte nucleus and the formation of true erythrocytes. Erythropoietin causes the erythrocyte colony-forming units to proliferate and mature.

She will have iron-deficiency anemia due to depletion of iron stores.

A 66-year-old female client has presented to the emergency department because of several months of intermittently bloody stool which has recently become worse. The woman has since been diagnosed with a gastrointestinal bleed secondary to overuse of nonsteroidal anti-inflammatory drugs that she takes for her arthritis. The health care team would realize that which situation is most likely? She will have iron-deficiency anemia due to depletion of iron stores. She will have delayed reticulocyte release. The client will be at risk for cardiovascular collapse or shock. The woman has depleted blood volume due to her ongoing blood loss

the client has possibly undergone damage as a result of calcification following cellular injury

A 68-year-old male client with aortic stenosis secondary to calcification of the aortic valve is receiving care. Which statement best captures an aspect of this client's condition? -Paget disease, cancer with metastases, or excess vitamin D may have been contributors. -The client has possibly exhibited phosphate retention leading to calcium deposits. -Increased calcium intake over time may have contributed to the problem. -The client has possibly undergone damage as a result of calcification following cellular injury.

Hemoglobin 9.9 g/dL

A 7-year-old boy is admitted to the hospital with a suspected diagnosis of lead toxicity. Which assessment finding is most congruent with the client's diagnosis? -White blood cells (WBC) 11,000/mm3 (11.0 x 109/L) -Diffuse muscle pain -Hemoglobin 9.9 g/dL (99 g/L) -Decreased deep tendon reflexes

Unsteady gait and difficulty speaking and swallowing

A 70 year-old male has been diagnosed with a stroke that resulted in an infarct to his cerebellum. Which clinical finding would be most closely associated with cerebellar insult? Unsteady gait and difficulty speaking and swallowing Flaccid loss of muscle tong Tremor, rigidity, and bradykinesia Difficulty starting movement, stopping movement, and maintaining rhythmic movements

You could remain the chronic stage of CML for several years before it accelerates and culminates in a crisis."

A 70-year-old woman has received a diagnosis of chronic myelogenous leukemia (CML) after a clinical investigation sparked by the presence of leukocytosis in her routine blood work. What clinical course should her care provider tell her to expect? "It's likely that this will give you chronic fatigue and malaise for the rest of your life, but that will probably be the extent of your symptoms." "You can expect your blood results, fatigue, and susceptibility to infection to gradually worsen over a few years." "Unfortunately, your leukemia will likely enter a crisis mode within a few weeks if we don't treat it immediately." "You could remain the chronic stage of CML for several years before it accelerates and culminates in a crisis."

Platelet aggregation can be precluded through inhibition of prostaglandin production by aspirin.

A 71-year-old male client with a history of myocardial infarction (MI) and peripheral vascular disease (PVD) has been advised by his family physician to begin taking 81 mg aspirin once daily. Which statement best captures an aspect of the underlying rationale for the physician's suggestion? Aspirin helps to inhibit ADP action and minimizes platelet plug formation. Aspirin inhibits the conversion of fibrinogen into fibrin and consequent platelet plug formation. Aspirin can reduce unwanted platelet adhesion by inhibiting TXA2 synthesis. Platelet aggregation can be precluded through inhibition of prostaglandin production by aspirin.

The portal circulatory system brings venous blood from the Gl tract into the liver.

A 77-year-old male client with a diagnosis of stomach cancer has been found to have metastases in his liver. The client and his family are surprised at this turn of events, stating that they don't see how he could have developed cancer in his liver. Which fact would underlie the reply that the care team provides? -The portal circulatory system brings venous blood from the Gl tract into the liver. -Hepatic stromal tissue shares characteristics with cancerous cells, including lack of anchorage dependence -The parenchymal tissue of the liver is particularly susceptible to secondary malignancies -The proximity of the liver to the stomach allows for direct spread of cancerous cells due to a lack of contact inhibition

The client notes that he has increased work of breathing when lying supine.

A 77-year-old man is a hospital inpatient admitted for exacerbation of his chronic obstructive pulmonary disease (COPD), and a respiratory therapist (RT) is assessing the client for the first time. Which aspect of the client's current state of health would be best characterized as a symptom rather than a sign? -The client's oxygen saturation is 83% by pulse oximetry. -The client notes that he has increased work of breathing when lying supine. -The client's respiratory rate is 31 breaths per minute. -The RT hears diminished breath sounds to the client's lower lung fields bilaterally

"Has your son had any sinus or ear infections in the last little while?"

A 9 year-old boy has been brought to the emergency department by his father who is concerned by his son's recent fever, stiff neck, pain, and nausea. Examination reveals a petechial rash. Which assessment question by the emergency room physician is most appropriate? 'Does your son have a history of cancer?" "Has your son had any sinus or ear infections in the last little while?" "Is your son currently taking any medications?" "Was your son born with any problems that affect his bone marrow or blood?"

This is a degenerative disorder that tends to be inherited as an autosomal recessive trait.!

A Toddler is displaying signs/symptoms of weakness and muscle atrophy The pediatric neurologist suspects it may be a lower or neuron disease called spinal muscular atrophy (SMA). The client's family asks how he got this. The clinician will respond: This could result from playing in soil and then ingesting bacteria that is how attaching his motor neurons." "This is a segmental demyelination disorder that affects all nerve roots and eventually all muscle groups as well! "This is a degenerative disorder that tends to be inherited as an autosomal recessive trait.! No one really knows how this disease is formed. We just know that in time, he may grow out of it."

"Your son has a contusion of the brain at the site where the bat hit his head."

A baseball player was hit in the head with a bat during practice. In the emergency department, the physician tells the family that he has a "coup' injury. How will the nurse explain this to the family so they can understand? 'It's like squeezing an orange so tight that the juice runs out of the top." "Your son has a contusion of the brain at the site where the bat hit his head." "Your son has a huge laceration inside his brain where the bat hit his skull. "When the bat hit his head, his neck jerked backward causing injury to the spine.!

Fibrinogen and clotting factor levels

A blood sample is ordered to be collected in a plasma-separator tube, but the nurse can only find serum-separator tubes available. The nurse should be most concerned about the accuracy of which results if the serum-separator tube is used? Albumin and total protein levels Electrolyte measurements Immunoglobulin analysis Fibrinogen and clotting factor levels

Frontal lobe Intra-axial Extra-axial

A brain tumor causing clinical manifestations of headache, nausea, projectile vomiting, and mental changes is likely located in which part of the brain? Select all that apply. Frontal lobe Intra-axial Extra-axial Temporal lobe Brain stem

Splenomegaly

A child has been diagnosed with thalassemia. Which of the following comorbidities may occur as a result of having thalassemia? Hypocoagulation Splenomegaly Iron deficiency Neutropenia

"This means the chemotherapy has lowered your blood counts including red blood cells, white blood cells and platelets. You are very sick, but we are working hard to make you healthy again."

A child with leukemia hears the health care provider tell the parents that the child has "pancytopenia." This child asks the nurse, "What does 'pancy' mean? Am I going to die?" Which response by the nurse is most appropriate for this 10-year-old child? "This means the chemotherapy has lowered your blood counts including red blood cells, white blood cells and platelets. You are very sick, but we are working hard to make you healthy again." "This is very serious and could possibly lead to death if you start bleeding badly. "This means the chemotherapy is attacking your bone marrow and your body has to find a way to fight back. No, you are not going to die." back "This is just a way of saying that you are sick and need to rest more. Your medicine is fighting your cancer cells so that you will not

Heart rate Blood pressure Time of symptom onset

A client arrives at the emergency department with symptoms of stroke. What evidence should the nurse gather to determine if the client is a candidate for thrombolytic therapy? Select all that apply. History of stroke LI Heart rate Age Blood pressure Time of symptom onset

prone, decubitus

A client arrives for a scheduled bone marrow aspiration and biopsy. The nurse explains to the client that which position(s) is best for accessing the posterior iliac crest for aspiration? Select all that apply. Trendelenburg Legs extended Supine Prone Lateral decubitus

The bruising around the ankles is due to the fact that it is a dependent area where the capillary pressure is higher.

A client asks the health care provider why his lower legs look purple. The health care provider will base the response on which pathophysiologic principle? There is a problem with the plasminogen levels. The bruising around the ankles is due to the fact that it is a dependent area where the capillary pressure is higher. Morbid obesity causes veins to enlarge and bleed into tissues due to stress the abdomen is placing on the vascular system. Too much trauma breaks capillaries and they bleed into the tissue.

Filgrastim, a granulocyte-colony stimulating factor (G-CSF)

A client at risk for febrile neutropenia related to chemotherapy treatments has been prescribed medication to prevent this reaction. Which medication should the nurse be prepared to administer? Prednisone, a corticosteroid Immunoglobulin M (IgM) antibody Epoetin alfa, an erythropoiesis-stimulating agents (ESAs) Filgrastim, a granulocyte-colony stimulating factor (G-CSF)

dexamethasone, a corticosteroid.

A client diagnosed with low-risk chronic lymphocytic leukemia (CLL) has recently developed thrombocytopenia. One of the medications utilized to treat this would be: cisplatin, a chemotherapeutic. dexamethasone, a corticosteroid. doxorubicin, a cytotoxic antibiotic. vincristine, a vinca alkaloid.

"They are looking for the presence of antibody or complement on the surface to the RBC."

A client has been diagnosed with anemia. The physician suspects an immune hemolytic anemia and orders a Coombs test. The client asks the nurse what this test will tell the doctor. The nurse replies: "They are looking for the presence of antibody or complement on the surface to the RBC." "They will look at your RBCs under a microscope to see if they have an irregular shape (poikilocytosis)." "They will be looking to see if you have enough ferritin in your blood." "They will wash your RBCs. Then mix the cells with a reagent to see if they clump together."

Avoid stressful situations. Stay up-to-date with your vaccines. Avoid extremes of environmental temperature

A client has been diagnosed with multiple sclerosis. What should the clinician include in client teaching to help reduce the frequency of exacerbations? Select all that apply. Stay out of direct sunlight. Do not take over-the-counter medications. Avoid stressful situations. Stay up-to-date with your vaccines. Avoid extremes of environmental temperature

RBC indices

A client has been suffering from fatigue and the health care provider suspects anemia. Which test results will the health care provider consult when investigating the cause of the anemia? RBC indices Hematocrit Red blood cell (RBC) Hemoglobin

gag reflex assessing for problems with chewing and swallowing

A client has experienced a pontine stroke which has resulted in severe hemiparesis. What priority assessment should the nurse perform prior to allowing the client to eat or drink from the food tray? ability to follow commands looking for hearing loss blood pressure reading looking for hypertension cornea blink reflex for ability to see the food tray gag reflex assessing for problems with chewing and swallowing

Immunohistochemistry

A client has stage IV cancer but further testing is needed to determine the site of origin of this metastatic tumor. Which form of testing will the health care provider be discussing with this client? -Papanicolaou test -immunohistochemistry -sentinal node recovery -microarray technology

Interferon beta alters your immune response. • The medication will be given by subcutaneous injection.

A client has started on interferon beta for treatment of multiple sclerosis. What should the clinician include in the teaching? Select all that apply. Interferon beta does not have any known side effects Interferon beta alters your immune response. • The medication will be given by subcutaneous injection. Seek emergency treatment if you develop flu-like symptoms This drug can reduce symptoms but will not alter disease activity.

vasogenic edema

A client in the intensive care unit who has a brain tumor has experienced a sharp decline. The care team suspects that water and protein have crossed the blood-brain barrier and been transferred from the vascular space into the client's interstitial space. Which diagnosis best captures this pathophysiology? Cytotoxic edema Vasogenic edema Focal hypoxia Hydrocephalus

Anticoagulation therapy

A client is admitted for cardioembolic stroke. Which therapy to best prevent recurrence of embolic stroke should the nurse monitor for effectiveness? Correction of dyslipidemia Blood pressure management Antiarrhythmic therapy Anticoagulation therapy

Blood pressure historically in the range of 150/90 to 160/100 mm Hg Diagnosed with type 2 diabetes 8 ears ago Black male

A client is admitted to a rehabilitation center after hospital treatment for an ischemic stroke. What aspects of the client's history are risk factors for ischemic stroke? Select all that apply. Takes corticosteroids for the treatment of rheumatoid arthritis Takes iron supplements for the treatment of chronic anemia Blood pressure historically in the range of 150/90 to 160/100 mm Hg Diagnosed with type 2 diabetes 8 ears ago Black male

A reduction of skeletal muscle use secondary to the traction treatment

A client is experiencing muscle atrophy following 2 weeks in traction after a motor vehicle accident. Which factor has most likely contributed to the atrophy of the client's muscle cells? -Denervation of the affected muscles during the time of traction -Reduced oxygen consumption and cellular function that ensures muscle cell survival -High levels of insulin and IGF-1 in the client's blood during immobilization -A reduction of skeletal muscle use secondary to the traction treatment

Anticholinergics

A client is having an edrophonium test. The client suddenly goes into respiratory arrest. Which priority medication should the nurse be prepared to administer? Anticholinergic Inhaled beta-2 agonist Acetylcholinesterase inhibitor Mixed alpha-beta adrenergic agonist

administer antibiotics

A client is suspected of having bacterial meningitis. Which action should the nurse complete first? Prepare for lumbar puncture Administer antipyretics Administer antibiotics Complete neurological assessment

"The TP53 gene is a group of tumor suppressor genes associated with lung, breast and colon cancer.!!

A client overheard the provider discussing the case and mentioning a "mutation in the TP53 gene./ The client asks the nurse, "What does that mean?" Which response is the most appropriate for this client? -The TP53 gene was likely damaged when you were in utero and exposed to a toxin." -All cancers result from an abnormality in some gene or chromosome." -"The TP53 gene is a group of tumor suppressor genes associated with lung, breast and colon cancer. -This means that they are looking for a monoclonal antibody that will selectively kill your tumor

unexplained pruritis night sweats

A client visited his health care provider after finding an enlarged lymph node along the mediastinal border. The health care provider may be alerted to a possible diagnosis of Hodgkin lymphoma (HL) based on the client having which other clinical manifestations? Select all that apply. Yeast infection in the mouth Unexplained pruritus Joint swelling Sore throat with pustules on tonsils. Night sweats

Some clients experience longer-term irritation of skin adjacent to the treatment site."

A client who has a diagnosis of lung cancer is scheduled to begin radiation treatment. The nurse knows that which statement about potential risks of radiation is most accurate? -"Sometimes you might find that your blood takes longer to clot than normal." -"The unwanted effects will be limited to the exposed portions of your skin." -"Some clients experience longer-term irritation of skin adjacent to the treatment site." -"The changes that you might see are normally irreversible."

A decrease in mean corpuscular volume

A client with a diagnosis of atrophic gastritis and consequent pernicious anemia is receiving high oral doses of vitamin B12. Following completion of his treatment, which lab finding demonstrates improvement in this condition? A decrease in mean corpuscular volume Decreased free heme levels Increased serum bilirubin Increased folic acid levels

"The hematocrit measures the mass that your red blood cells account for in a quantity of your blood."

A client with a diagnosis of hemolytic anemia has gone to a community-based laboratory for follow-up blood work. The lab technician confirms with the client that hematocrit is one of the components of the blood work. The client replies, "I thought the point of the blood work was to see how many red blood cells I have today." How could the technician best respond to the client's statement? "Your hematocrit measures the average size of your red blood cells, and indirectly measures your oxygen-carrying capacity." "The hematocrit measures the mass that your red blood cells account for in a quantity of your blood." "This result will tell your care provider about the number of red blood cells in a given quantity of your blood plasma." "The result will indicate how many of your red blood cells are new and young, and will indicate your body's production rate of red cells."

A-

A client with a gastrointestinal bleed secondary to alcohol abuse and a hemoglobin level of 5.8 g/dL (58 g/L) has been prescribed a transfusion of packed red blood cells. The client possesses type B antibodies but lacks type D antigens on his red cells. Transfusion of which blood type would be least likely to produce a transfusion reaction? A- B- A+ B+

recent medication history

A client with a history of epilepsy is brought to the emergency department due to seizure activity. What information about the client is most important for the nurse to collect? Recent medication history Allergies to medications Family history of seizure disorders Type of seizure activity experienced

transfuse 2 units of platelets transfuse fresh frozen plasma

A client with leukemia begins hemorrhaging from every orifice. The physician is concerned that the client has developed disseminated intravascular coagulation (DIC). The nurse should anticipate which order to be prescribed for this client? Select all that apply. Give aspirin twice/day Transfuse 2 units of platelets Place in reverse isolation. Transfuse fresh frozen plasma. Administer IV ketorolac STAT.

fever, chills, fatigue

A client with malignant melanoma has been prescribed alpha interferon, a biologic response modifier. Since this drug prolongs the cell cycle, increasing the percentage of cells in the Go phase, and stimulates NK cells and T-lymphocyte killer cells, the nurse can anticipate that they may experience which common side effects? •Renal damage with an increased creatinine level • Nausea, vomiting, and diarrhea • Opportunistic infections like Candida ) •Fever, chills, and fatigue

liver, spleen

A client with severe anemia asks, "What happens if my bone marrow cannot produce sufficient numbers of red blood cells due to my bleeding?" Which response(s) by the nurse correctly identifies the organs that can resume hematopoiesis? Select all that apply. Gall bladder Liver Spleen Kidneys Pancreas

dysphagia, vision changes, unilateral weakness

A clinical nurse specialist is interested in developing a research study focused on clients living with the sequelae of ischemic stroke. Which clients should the nurse include? Select all that apply. The nurse should include clients with: aspiration pneumonia. -dysphagia (difficulty swallowing). -falls -vision changes. -unilateral weakness.

A tall, thin, myopic, 28-year old woman with mitral valve prolapse

A clinician who works on a cardiac care unit of a hospital is providing care for a number of clients. Which client most likely has a genetic disorder arising from inheritance of a single gene? •A tall, thin, myopic, 28-year old woman with mitral valve prolapse • A short, thin, 56-year-old woman with hypertension •An overweight, middle-aged male smoker with coronary artery disease •A thin, middle-aged nonsmoking man with a repaired atrial septal defect

I'm going to tap (percuss) over the median nerve in your wrist; tell me what sensation you feel while I am doing this. Does the sensation stay in the wrist or go anywhere else?

A clinician working in a busy orthopedic clinic is asked to perform the Tinel sign on a client having problems in her hand/wrist. In order to test Tinel sign, the clinician should give the client which direction? Hold your wrist in complete flexion, keep it in this position for 60 seconds. How does your hand feel after placing it in a neutral position? I'm going to tap (percuss) over the median nerve in your wrist; tell me what sensation you feel while I am doing this. Does the sensation stay in the wrist or go anywhere else? I'm going to tap this tuning fork, place it on the side of your thumb. then tell me what you are feeling in your hand and wrist. Stand tall, arms at your side, shut your eyes; place the tip of your index finger to your nose.

High pressure and local hemorrhage of the venous system

A college student has been experiencing frequent headaches that he describes as throbbing; he is reporting difficulty concentrating while studying. Upon cerebral angiography, he is found to have an arteriovenous malformation. Which pathophysiologic concept is likely responsible for his symptoms? Hydrocephalus and protein in the cerebrospinal fluid Increased tissue perfusion at the site of the malformation 7 • High pressure and local hemorrhage of the venous system Localized ischemia with areas of necrosis noted on CT angiography

"If you choose to feed your baby with formula, ensure that it is iron-fortified."

A community health nurse is conducting the nutritional component of a class for new mothers. Which teaching point would be most justified? "Iron supplementation is not necessary provided you are breast-feeding your infant." "If you choose to feed your baby with formula, ensure that it is iron-fortified." "Your infant needs the same amount of iron as you, but has far fewer sources for obtaining it." "Be aware that cow's milk depletes your baby's supply of iron."

The client has a family history of cardiovascular disease related to hypercholesterolemia and remains noncompliant with the treatment regime.

A community health nurse is teaching a group of recent graduates about the large variety of factors that influence an individual's health or lack thereof. The nurse is referring to the Healthy People 2020 report from the U.S. Department of Health and Human Services as a teaching example. Of the following aspects discussed, which would be considered a determinant of health that is outside the focus of this report? -The client has a good career with exceptional preventive health care benefits. -The client has a diverse background by being of Asian and Native American/First Nation descent and practices various alternative therapies to minimize effects of stress. -The client lives in an affluent, clean, suburban community with access to many health care facilities. -The client has a family history of cardiovascular disease related to hypercholesterolemia and remains noncompliant with the treatment regime.

"Testing the umbilical blood and performing amniocentesis can give us some information, but not a guarantee."

A couple who are pregnant with their first child have made an appointment with a clinical geneticist to discuss prenatal screening. The man states that they, "just want to make sure that there is nothing wrong with our baby." How could the clinician best respond to this statement? -We can't rule out all abnormalities, but a routine fetal tissue biopsy can yield useful information! -Prenatal screening is not usually necessary unless you are among a high-risk group. -You need to be aware that if abnormalities are detected, termination is normally required! -"Testing the umbilical blood and performing amniocentesis can give us some information, but not a guarantee."

Completed chemotherapy for leukemia in the past year Recent bout with mononucleosis viral infection White blood cell (WBC) count <1,000/μL (1 ×109/L) Platelet count 50 ×103/μL (50 ×109/L)

A female client arrives at the clinic reporting fatigue that is exhausting, bruising on the skin, and bleeding from the gums. Which diagnostic result(s) or history finding(s) correlates with a diagnosis of aplastic anemia? Select all that apply. Hemoglobin level of 12.5 g/dL (125 g/L) Completed chemotherapy for leukemia in the past year Recent bout with mononucleosis viral infection White blood cell (WBC) count <1,000/μL (1 ×109/L) Platelet count 50 ×103/μL (50 ×109/L)

5-6 liters of blood

A fifth-grade elementary student asks the school nurse how much blood is in an entire body. The nurse should respond that the average grown-up adult has: 3 pints of blood total 3 to 4 quarts of blood in the body 2 to 4 cups of blood in the bod 5 to 6 liters of blood throughout the body

lupus

A geriatrician is following a number of clients on a subacute geriatric medical unit, some of whom require diagnostic blood work. Which client would be most likely to have an erythrocyte sedimentation rate (ES) screening test ordered? An adult with: orthostatic hypotension and syncopal episodes. congestive heart failure. systemic lupus erythematosus. a diagnosis of Alzheimer disease and depression.

an enzyme defect causing abnormal lipid accumulation in the brain

A home health nurse is making a visit to a family with an 8-month-old infant with severe motor deterioration. The physician has diagnosed the infant with Tay-Sachs disease. The parents are asking the nurse why this happened. The nurse will base her answer knowing the root cause of Tay-Sachs is: -high exposure to lead in the home environment. -an increase in bilirubin retention leading to destruction of RBCs. -an enzyme defect causing abnormal lipid accumulation in the brain. -hypoxia caused by placing the infant on his or her abdomen during sleep.

Suppression of fibrin formation

A hospital client is receiving intravenous infusion of heparin for treatment of a pulmonary embolus. Which phenomenon is most likely occurring, resulting in the drug's therapeutic effect? Suppression of fibrin formation Inhibition of ADP-induced platelet aggregation Deactivation of the intrinsic clotting pathway Inhibition of vitamin K synthesis in the liver

The bottom layer of blood in the tube accounts for around 40% to 45% of the total volume.

A hospital laboratory technician performing routine blood analysis as part of an inpatient's assessment is examining the sample in a test tube following processing in a centrifuge and the addition of an anticoagulant. Which observation would the technician most likely interpret as an anomaly? The bottom layer of blood in the tube accounts for around 40% to 45% of the total volume. The yellowish fluid on the top of the sample appears to constitute around one-quarter of the total volume. The middle layer of cells appears white to grey in color. The top laver of cells is too thin to visualize without microscopy.

Tall stature out of proportion Sparse facial and pubic hair enlarged breast tissue

A male child was born with Klinefelter syndrome. As an adolescent, the nurse will assess which clinical manifestations of Klinefelter syndrome? Select all that apply. •Severe intellectual disability •Enlarged breast tissue •Higher than average linguistic skills • Tall stature out of proportion •Sparse facial and pubic hair

"I know that a single mutant allele is to blame for the health problem."

A male client has an autosomal dominant disorder. The client and his partner are considering starting a family. Which statement indicates the client has an adequate understanding of the genetic basis of this health problem? -"I know that new genetic mutations won't occur between generations." -"I know there's no way of accurately determining the chance that my child will inherit the disease." -"I know that a single mutant allele is to blame for the health problem." -"My children who don't have the disease still run the risk of passing it on to their children."

the preclinical stage of disease

A male international business traveler has returned from a trip to Indonesia. While there, he hired a prostitute for companionship and engaged in unprotected sex on more than one occasion. Unbeknownst to him, this prostitute harbored the hepatitis C virus. Upon return to the U.S., he exhibited no symptoms and returned to his usual activities. During this period of no outward symptoms, the disease would be classified as being in: -the preclinical stage of disease. -remission and unlikely to develop hepatitis C. -the clinical disease stage of hepatitis C. -the chronic phase of hepatitis C.

A 21-year-old male with a diagnosis of cellulitis and suspected endocarditis secondary to intravenous drug use

A medical student is familiarizing herself with recent overnight admissions to an acute medical unit of a university hospital. Which client would the student recognize as least likely to have a diagnosis of antiphospholipid syndrome in his or her medical history? A 90-year-old female resident of a long-term care facility who has been experiencing transient ischemic attacks A 21-year-old male with a diagnosis of cellulitis and suspected endocarditis secondary to intravenous drug use A 66-year-old obese male with left-sided hemiplegia secondary to a cerebrovascular accident A 30-year-old female with a diagnosis of left leg deep vein thrombosis and a pulmonary embolism

A client who is has increasing serum ammonia levels due to liver cirrhosis

A member of the health care team is researching the etiology and pathogenesis of a number of clients who are under his care in a hospital context. Which aspect of clients' situations best characterizes pathogenesis rather than etiology? -A client with multiple skeletal injuries secondary to a motor vehicle accident. -A client who was admitted with the effects of methyl alcohol poisoning. cirrhosis. -A client who is has increasing serum ammonia levels due to liver cirrhosis -A client who has been exposed to the mycobacterium tuberculosis bacterium.

Cerebellar damage caused by a cerebrovascular accident

A middle-aged woman is brought to the emergency room after a minor auto accident. Her gait is staggering and unsteady, her speech is slurred, and she displays slight nystagmus. The police officer who brought her in says she has not been drinking. Her blood pressure is very high. Which health problem most likely underlies her present state? Guillain-Barre syndrome Cerebellar damage caused by a cerebrovascular accident Myasthenia crisis Multiple sclerosis

Staffing a booth where community residents who are attending a baseball tournament can have their blood glucose levels checked.

A multidisciplinary healthcare team operates a program aimed at the prevention, identification, and treatment of diabetes on a large Indian reservation. Which aspect of the program would most likely be classified as secondary prevention? -Staffing a booth where community residents who are attending a baseball tournament can have their blood glucose levels checked. -Regularly scheduled wound dressing changes for clients who have foot ulcers secondary to peripheral neuropathy and impaired wound healing. -Administering oral antihyperglycemic medications to clients who have a diagnosis of diabetes -Teaching school children how a nutritious, traditional diet can lessen their chances of developing adult-onset diabetes.

"Sounds like this began because of inflammation inside my artery that made it easy to form fatty streaks which led to my clogged artery."

A new client who suffered a myocardial infarction requires angioplasty and stent placement. He has arrived to his first cardiac rehabilitation appointment. In this first session, a review of the pathogenesis of coronary artery disease is addressed. Which statement by the client verifies to the nurse that he has understood the nurse's teachings about coronary artery disease? -"Sounds like this began because of inflammation inside my artery that made it easy to form fatty streaks which led to my clogged artery." -" If vou do not exercise regularly to get your heart rate up. blood pools in the veins causing a clot which stops blood flow to the muscle and you have a heart attack." -"My artery was clogged by fat so I will need to stop eating fatty foods like french fries every day' lack -"All l have to do is stop smoking and then I won't have any more heart attacks"

"Her diagnosis puts her at higher risk of developing a malignant neoplasm."

A new older female client at a long term care facility has a diagnosis of neurofibromatosis type 1. As part of the intake assessment protocol for the facility, the clinical educator is teaching the care staff about the diagnosis. Which statement most accurately conveys an aspect of neurofibromatosis? -"Her diagnosis puts her at higher risk of developing a malignant neoplasm." -"The neurofibroma lesions are unsightly for the client. but they are not painful! -"She is living with an example of an autosomal recessive disorder. -The client is likely to be photosensitive as a result of the disease!

TIAs resolve rapidly but may place the client at an increased risk for stroke.

A nurse at a long term care facility provides care for an 85 year-old man who has had recent transient ischemic attacks (TIAs). Which statement best identifies future complications associated with TIAs? TAs are a relatively benign sign that necessitate monitoring but not treatment. TIAs are caused by small bleds that can be a warning sign of an impending stroke TIAs resolve rapidly but may place the client at an increased risk for stroke. TIAs are an accumulation of small deficits that may eventually equal the effects of a full CVA

decreased blood flow has induced hypoxia

A nurse in the emergency department admits a male client who has experienced severe frostbite to his hands and toes after becoming lost on a ski hill. The nurse recognizes that which phenomena has contributed to his tissue damage? -Autonomic nervous stimulation has resulted in injury. -Decreased blood viscosity has resulted in interstitial bleeding. -Decreased blood flow has induced hypoxia -.Reactive vasodilation has compromised perfusion.

be asymptomatic at an early stage

A nurse is considering setting up a screening program for a specific health condition in a population. What characteristic of the condition would need to be true for the nurse to justify screening a population? The condition should: • be asymptomatic at an early stage. •have a high mortality rate. •be curable with available treatment •have a noninvasive diagnostic test available

-The geographic location of women diagnosed with breast cancer -The age of women at the time of diagnosis

A nurse is investigating the epidemiologic factors influencing breast cancer for women in a population. What information should the nurse include? Select all that apply. -The geographic location of women diagnosed with breast cancer -The currently known incidence and prevalence for this population -The age of women at the time of diagnosis -The number of women whose diagnosis was assisted by the use of mammography How cancer cells divide and multiply in breast tissue

antioxidants inhibit the actions of reactive oxygen species

A nurse is teaching a group of older adults about the value of including foods containing antioxidants in their diet. Which statement best captures the rationale underlying the nurse's advice? -Antioxidants prevent the occurrence of cell dysplasia. -Antioxidants react nonspecifically with molecules. -Antioxidants prevent the formation of superoxide dismutase. -Antioxidants inhibit the actions of reactive oxygen species.

Burn unit

A nurse is triaging clients at a disaster site. Local facilities have different specialized units. To what facility should the nurse send a client who has sustained an electrical injury to his left thigh? -Burn unit -Surgical intensive care unit -Cardiac care unit -Neurology unit

coma

A nurse on a neurology unit is assessing a female brain-injured client. The client is unresponsive to speech, and her pupils are dilated and do not react to light. She is breathing regularly but her respiratory rate is 45 breaths per minute. In response to a noxious stimulus, her arms and legs extend rigidly. What is her level of impairment? O Delirium Coma Vegetative state Brain death

infections

A nurse practitioner is providing care for a client with low levels of the plasma protein gamma globulin. The nurse would recognize that the client is at risk of developing which health problem? Jaundice • Anemia • Blood clots • Infections

cohort

A nurse practitioner is working in a crowded neighborhood where the population is primarily immigrants from China. The nurse has designed a research study to follow children from kindergarten to the age of 25. She is going to be looking at their diet, successful progression in school, health practices, and development of disease, to name a few items. This type of research is known as: •Cohort study •Case-control study • Cross-sectional study •Epidemiologic study

Focusing on clients who did not receive treatment for the condition

A nurse researcher is interested in the natural history of a disease being studied. What should the nurse focus on to best understand the condition's natural history? -Examining outcomes across a wide variety of treatment approaches -Focusing on clients who did not receive treatment for the condition -Interviewing clients for their first-hand experience with the condition -Following client cases from initial diagnosis and throughout treatment

high morbidity, low mortality, high prevalence, low incidence

A particular disease has a debilitating effect on the ability of sufferers to perform their activities of daily living, and is a significant cause of decreased quality of life. However, few people die as a result of the disease's direct effects. There are hundreds of thousands of Americans living with the disease but relatively few new cases in recent years. Which statement best conveys an accurate epidemiologic characterization of the disease? -High morbidity; low mortality; high prevalence, low incidence. -High mortality: low morbidity; high incidence; low prevalence -Low mortality; high morbidity; low prevalence: high incidence. -Low mortality high morbidity; high incidence; low prevalence.

CHD

A physician is working with a 30-year-old male client with Down syndrome who has been admitted to hospital with a diagnosis of acute leukemia. Which physical assessment finding would the physician be more likely to find in an examination of this client than other clients without Down syndrome? •Decreased visual acuity •Diabetes mellitus •Hepatomegaly • Congenital heart defects

All of the tests used to examine fetal DNA are invasive and carry associated risks. An ultrasound will be required for either amniocentesis or chorionic villus sampling.

A pregnant client is exploring options related to prenatal testing due to a history of inherited disorders. What information should the nurse include when informing the client about various prenatal tests? Select all that apply. •The type of test used is determined by your preference and level of invasiveness •All of the tests used to examine fetal DNA are invasive and carry associated risks. •Percutaneous umbilical cord blood sampling is needed for a truly accurate result. •An ultrasound will be required for either amniocentesis or chorionic villus sampling. •If genetic disorders are being investigated, chorionic villus sampling will be required

Males are less likely to be born with fragile X syndrome. Daughters are less severely affected than sons by fragile X syndrome. You have a 50% chance of passing the condition to a child of either sex.

A pregnant client learns she has fragile X syndrome through prenatal screening and asks how this might affect the unborn child. What information should the nurse include in the response? Select all that apply. -If your husband is not a carrier, your children will not have fragile X syndrome. -You have a 50% chance of passing the condition to a child of either sex. -If you have a daughter, there is a 100% chance she will have the condition. -Males are less likely to be born with fragile X syndrome. -Daughters are less severely affected than sons by fragile X syndrome.

The nurse evaluates the ECG of a client who has developed hypokalemia secondary to diuretic use. Which manifestation of hypokalemia does the nurse anticipate on the ECG?

A prominent U wave and flattened T wave

Football hockey

A school nurse is working with a 16-year-old client recovering from mononucleosis. Which activities should the nurse recommend the teenager avoid while recovering? Select all that apply. Driving a vehicle Tennis Football Swimming Hockey

"Actually, plasma plays a significant role in nutrient and waste transport."

A student makes the statement to a colleague, "Blood plasma is essentially just a carrier for the formed cells like red blood cells and white blood cells. What would be the most accurate response to this statement? "Actually, plasma plays a significant role in nutrient and waste transport." "That's not really true. Plasma is crucial in the immune and inflammatory responses." "Actually, plasma is integral to the proper function of the liver and maintenance of acid-base balance: "Not really. Plasma also contributes to the processes of protein synthesis and hematopoiesis.""

"Your son's spleen is holding on to too many of his platelets so they're not available for clotting."

A surgeon is explaining to the parents of a 6-year-old boy the rationale for the suggestion of removing the boy's spleen. Which teaching point would be most accurate? We think that his spleen is inhibiting the production of platelets by his bone marrow! We belice that your son's spleen is causing the destruction of many of his blood platelets, putting him at a bleeding risk. "Your son's spleen is holding on to too many of his platelets so they're not available for clotting." "Your son's spleen is inappropriately destroying the platelets from his blood and needs to be removed.

encephalitis

A teenager, exposed to West Nile virus a few weeks ago while camping with friends, is admitted with headache, fever, and nuchal rigidity. The teenager is also displaying some lethargy and disorientation. The nurse knows which medical diagnosis listed below may be associated with these clinical manifestations? Spinal infection • Encephalitis 7 Rocky mountain spotted feve •Lyme disease

chronic alcohol use

A woman gives birth to a small infant with a malformed skull. The infant grows abnormally slowly and shows signs of substantial cognitive and intellectual deficits. The child also has facial abnormalities that become more striking as it develops. What might you expect to find in the mother's pregnancy history? -Chronic cocaine use -Active herpes simplex infection -Chronic alcohol use -Folic acid deficiency

Most mercury toxicity involves central nervous system changes.

A worker in a warehouse is trying to have children but think he or she has handled "mercury" while cleaning equipment. Which statement by the occupational nurse is most appropriate at this time? -You should have all the mercury fillings in your teeth changed to newer products. -Usually mercury toxicity appears as skin rashes and lesions. -Most mercury toxicity involves central nervous system changes. -Infertility is caused by too much mercury exposure.

the best time to allow the child to play outside in the evening hours, wear long sleeves, long pants, gloves, a hat, sunglasses

A young client has just been diagnosed with xeroderma pigmentosum. When teaching this family about this disease, the nurse should emphasize which teaching points? Select all that apply. -The best time to allow the child to play outside is in the evening hours after the sun goes down. -Wash hands thoroughly when working in the garden to prevent infection. -Apply antibacterial ointment to any break in the skin and cover wounds with bandages. -The best time for this family to go to the beach is in the fall/winter months. -Wear long sleeves, long pants, gloves, a hat, sunglasses with side shields, and sunscreen while outdoors.

"These hemorrhages are called petechiae and occur when platelets are deficit."

After assessment, the nurse asks how long the client has had red, pinpoint hemorrhages on the lower legs. The client responds, "This is the first time I have noticed this. What is wrong with me that is causing these small hemorrhages?" Which response by the nurse is most accurate? "Your platelets are developing a deformity as they are being produced by the bone marrow." "These hemorrhages are called petechiae and occur when platelets are deficit." "More than likely, you bumped something and these are a result of trauma to the vessel." You might have a pooling of all your platelets in the spleen. I will assess for that next."

paraplegia

After being thrown off the back of a bull, a bull rider can move his arms but has loss of motor function in the lumbar and sacral segments of the spinal cord. This is referred to as Anterior cord syndrome Tetraplegia Quadriplegia Paraplegia

A 45 year old woman with a diagnosis of shingles is experiencing an acute onset of severe neuropathic pain. Which stage of Selye's characterization of stress response is the woman most likely experiencing at the moment?

Alarm

Our son or daughter likely won't show the effects of sickling until he or she is school-aged because of the different hemoglobin in babies."

Amniocentesis has suggested that a couple's first child will be born with sickle cell disease. The parents are unfamiliar with the health problem and their caregiver is explaining the complexities. Which statement by the parents would suggest a need for further teaching or clarification? "Sickled cells can block his or her blood vessels, especially in the abdomen, chest, and bones." "Our son or daughter likely won't show the effects of sickling until he or she is school-aged because of the different hemoglobin in babies." "Not all of his or her red cells will be sickled, but low oxygen levels can cause them to become so." "Our baby's red cells are prone to early destruction because of their weak membranes."

echo

An 11-year-old girl is suspected of having Turner syndrome. Which diagnostic test would be the most useful component of screening to confirm or rule out the diagnosis? • Echocardiogram •Liver biopsy •Bone scan •Computed tomography of the head

Monitoring for any type of bleeding.

An adult female, newly diagnosed with possible leukemia, arrives for a bone marrow biopsy. The admission laboratory work includes: red blood cell count (RBC): 3.4 ×10/ML (3.4 ×1012/L); white blood cell count (WBC): 2,500/ L (2.5 ×10° /L); platelet count of 80 ×103/L (80 ×10°/L); all below normal values. What assessment is the highest priority for this client? Assessing heart sounds for any new murmurs. Monitoring for any type of bleeding. Checking breath sounds twice a day for signs of pneumonia Forcing fluids to maintain renal function.

case- control study

An epidemiologist is conducting a program of research aimed at identifying factors associated with incidence and prevalence of congenital cardiac defects in infants. The researcher has recruited a large number of mothers whose infants were born with cardiac defects as well as mothers whose infants were born with healthy hearts. The researcher is comparing the nutritional habits of all the mothers while their babies were in utero. Which type of study is the epidemiologist most likely conducting? • Cohort study -Cross-sectional study • Case-control study -Risk factor study

tertiary prevention

An occupational therapist conducts a group therapy program called MindWorks with older adults who have diagnoses of dementia and Alzheimer's disease. The goal of the group is to slow the cognitive decline of clients by engaging them in regular, organized mental activity such as reading maps and solving puzzles. How would the program most likely be characterized? •Secondary prevention • Prognosis enhancement •Primary prevention • Tertiary prevention

There will be a greater number of cancers that are epithelial in origin.

An oncology nurse who has worked for many years providing care for children with cancer has taken a job on an adult oncology unit of a hospital. What differences might the nurse anticipate in this new job? -More clients will be receiving treatment for leukemia -The nurse will be working with more clients who have blastomas. -There will be a greater number of cancers that are epithelial in origin. -A greater proportion of the clients will have cancer that involves the hematopoietic system.

The nurse knows high incidences of infectious illnesses among the older adults who reside in a long-term care facility are most likely to have diminished immune capacity because of: A. decreased numbers and responsiveness of T lymphocytes. B. decreased antigen recognition by B lymphocytes. C. over-expression of cytokines and receptors. D. altered function in peripheral lymphocytes.

Answer: A Rationale: Although this phenomenon is not well understood, increasing proportions of lymphocytes become unresponsive with age, and CD4+ T lymphocytes are the most severely affected. B lymphocytes recognize more antigens, not fewer, and expression of cytokines and their cellular receptors decreases.

Which scenario best describes an example of infection originating with a fomite? A. A client who contracted hepatitis C through sharing a contaminated syringe with an infected person B. A nurse with a positive tuberculin screening test (PPD) after admitting a client diagnosed with tuberculosis (TB) C. A woman who contracted Lyme disease after a tick bite D. A man who has been diagnosed with trichinosis after eating undercooked pork

Answer: A Rationale: An object that carries an infectious organism, such as a dirty syringe, is known as a fomite. Airborne contact with TB does not utilize a fomite. An infection via contact with an infected arthropod constitutes zoonosis. A person who consumes ova in undercooked meat acquires an infection by ingestion.

The nurse knows which statement listed below is accurate regarding the functions and nature of cytokines relative to a variety of pathologies? A. "A particular cytokine can have varied effects on different systems, a fact which limits their therapeutic use." B. "Cytokine production is constant over time, but effects are noted when serum levels cross a particular threshold." C. "Most cytokines are produced by granular leukocytes, and different cells are capable of producing the same cytokine." D. "Cytokine actions are self-limiting, in that activation of one precludes activation of other cytokines with similar actions."

Answer: A Rationale: Because cytokines can mediate diverse effects due to their pleotropic properties, they can have significant side effects. Cytokine production is brief, not constant, and production does not normally take place in granulocytes. Activation of a cytokine does not necessarily limit other similar cytokines, and additive effects are not uncommon.

The nurse is caring for a client who has just been diagnosed with shingles. The client states, "I have not been around anyone with this condition. How could I have caught it?" What is the nurse's best response? A. "This is a latent virus that you likely were exposed to as a child and it has now become active in the form of shingles." B. "This virus has a long prodromal stage so you could have been exposed over a month ago and not been aware." C. "Shingles is caused by recent exposure to someone who has active herpes zoster virus or has just been vaccinated." D. "This virus is the same one that causes cold sores and is manifesting in a different way this time, so it is very common."

Answer: A Rationale: Chickenpox and zoster (varicella-zoster or herpes-zoster, also known as shingles) are caused by the same virus. It has the initial manifestation as chickenpox but the resumption of the latent virus manifests as shingles. The client would not have required recent exposure. Herpes zoster is a latent virus rather than having a long prodromal stage. Cold sores are caused by herpes simplex virus I, not zoster.

The nurse will most likely assess which clinical manifestation in a client diagnosed with Creutzfeldt-Jakob disease? A. Change in behavior and memory, loss of coordination leading to encephalopathy B. Gastrointestinal problems like vomiting and diarrhea C. Muscle inflammation and edema, making movements very painful D. Projectile vomiting, hypertension, and drowsiness caused by elevated ICP

Answer: A Rationale: Creutzfeldt-Jakob disease is known collectively as a transmissible neurodegenerative disease that is characterized by a slowly progressive, noninflammatory, neuronal degeneration and leads to a loss of coordination, dementia and death. With this disease, encephalopathy is the primary presenting factor. GI problems, muscle inflammation, and ICP are not clinical manifestations associated with this prion.

Which phenomena best accounts for the increased presence of leukocytes at the site of inflammation? A. Existing leukocytes stick to the epithelial cells and move along blood vessel walls. B. Increased numbers of leukocytes are released into circulation via cytokine stimulation. C. Leukocytes are osmotically drawn from circulation into the interstitial space as a result of swelling. D. Epithelium expresses leukocyte stimulation factors in response to cell injury.

Answer: A Rationale: During inflammation, leukocytes accumulate at the point of epithelial contact in the processes of margination, adhesion, and transmigration. This is not directly achieved by way of increased leukocyte production or release, nor by osmotic pressure. The epithelium does not produce leukocyte stimulation factors.

The nurse is caring for a group of postoperative clients. Who should the nurse prioritize as having the greatest risk for infection at the surgical site? A. A 45-year-old with external fixation of a fractured tibia B. A 15-year-old 1 day post-tonsillectomy C. A 65-year-old awaiting pacemaker insertion D. A 22-year-old 2 days post-appendectomy

Answer: A Rationale: External fixation involves surgical pins that stabilize the bone and exit the body to be attached to a frame. Because these pins offer a route for pathogen entry—and are themselves foreign bodies—this client should be regarded as the highest risk compared to the other clients. A 65-year-old may have a less robust immune system, but in this case, the client has not yet undergone an invasive PRIMEXAM.COM procedure. The client post-appendectomy has no foreign body and will have the wound approximated via sutures, which will protect from infection. The client post-tonsillectomy is also at low risk. There is no external wound; additionally, there is protective antimicrobial action saliva and the rapid division of the mucosal epithelium at the site of surgery.

A nurse is conducting an immunization clinic when a 14-year-old client asks, "Does this vaccine make me immune to the disease?" What is the nurse's best response? A. "The vaccine imitates a disease without causing illness, resulting in the creation of antibodies to the disease." B. "The vaccine introduces substances that replicate and can kill the microorganisms if you are exposed." C. "The vaccine contains medicines that are designed to combat the microorganisms that cause this disease." D. "The vaccine will not make you immune but will improve your chances of surviving an infection."

Answer: A Rationale: Immunization with a vaccine triggers active immunity in which the person's own immune system creates antibodies to the microorganism introduced through the vaccine. The vaccine does not kill the microorganisms or contain "medicines" that combat the disease. Some vaccines can confer immunity. There is no indication in the scenario that the disease being vaccinated for has the potential to be fatal.

Which individual situation listed below best exemplifies the processes of innate immunity? A. A child who has experienced heat and swelling of his skinned knuckle B. An adult who complains of itching and is sneezing because he is allergic to pollen C. A client whose blood work indicates increased antibody titers during an acute illness D. A client who has experienced rejection of a donor liver after transplantation

Answer: A Rationale: In a skinned knuckle, one of the body's main innate defenses—the skin— is breached. The heat and swelling that accompany a breach in the skin are inflammatory responses, part of the body's innate immune defenses. Allergies are an inappropriate adaptive response mediated by immunoglobulin E; antibody titers increase during illness in response to the infection; and transplanted organs are rejected because the organ is recognized as foreign. These are all aspects of specific, acquired immunity.

The nurse is preparing a client for bone marrow transplant. The client asks, "How long will it take for the new marrow to make the blood cells?" What is the nurse's best response? A. Although it can vary, we will administer medication that will stimulate the bone marrow to create the cells. B. The stem cells that are injected will take the normal amount of time to create the cells as if it were your own marrow. C. The new marrow will not be creating blood cells until it has homed to the appropriate areas and matured. D. Because we place the donated marrow directly into your iliac crest, the marrow will start making cells right away.

Answer: A Rationale: In clinical practice, recombinant colony-stimulating factors are used to increase the success rates of bone marrow transplantations. The medication would be used to speed up the creation of red and white blood cells, and perhaps for platelets as well, rather than simply waiting for the "normal time" this would take. The donated marrow does not need to "mature" and it is injected intravenously, not into the client's iliac crest.

A nurse is providing care for a client who is immunocompromised following chemotherapy. The nurse knows that which characterization of the adaptive immune system is responsible for the client's disruption in normal immune function? A. Epitopes on antigens are recognized by immunoglobulin receptors following presentation by accessory cells. B. Haptens combine to form epitopes that stimulate the response of regulatory and effector cells. C. Effector cells orchestrate the immune response of regulatory cells toward an antigen. D. Accessory cells such as macrophages are engulfed by regulatory cells, stimulating effector cells.

Answer: A Rationale: In the adaptive immune response, accessory cells present antigen epitopes to receptors, initiating the immune response of lymphocytes. Epitopes may combine to form haptens, and regulatory cells orchestrate effector cells. Regulatory cells do not engulf accessory cells.

The nurse is caring for a client with a diabetic foot ulcer who has difficulty adhering to dietary restrictions. This has resulted in elevated blood glucose levels. What information is most important for the nurse to share with the client? A. Elevations in blood glucose make it more difficult for your white blood cells to fight the infection. B. We can have the registered dietitian come to speak to you about the importance of following your diet. C. High blood glucose levels damage your arteries and reduce the blood flow to your wound. D. When you do not comply with your diet, it makes it more difficult for us to help you get well.

Answer: A Rationale: In this case, the nurse should focus on the immediate consequences of the client's choices. In the short-term, hyperglycemia diminishes chemotactic and phagocytic function, including engulfment and intracellular killing of bacteria. The changes related to peripheral artery disease and diminished blood flow to the wound are also a factor, but these changes take years to occur; conversely, even short-term elevations of blood glucose impair phagocytosis, so this information is more relevant to the client's situation with an active infection. The nurse should not refer the client to a dietitian until first discussing the immediate consequence of the current choices. Finally, the nurse should not shift the focus to the health care providers by telling the client the choices make their job more difficult.

The nurse is caring for a client who meets the physical assessment criteria for sepsis, but the specimen cultures are negative for a causative organism. What action should the nurse take next? A. Administer the prescribed antibiotics. B. Recollect specimen cultures. C. Contact the health care provider. D. Continue to monitor signs and symptoms.

Answer: A Rationale: In this scenario, the specimens for culture had already been collected, so antibiotics should be given. The causative organism cannot be identified by culture in up to 33% of clients presenting with sepsis, so treatment should not be delayed for this reason. If the antibiotics are already prescribed, there is no need to contact the health care provider. Delaying antibiotic therapy will increase the risk for client mortality.

A 10 year old child with strep throat asks the nurse, "Why are there large bumps [lymph nodes] on my neck when my throat gets sore?" The nurse replies that lymph nodes: A. help your body fight off infections by allowing special cells [lymphocytes and macrophages] to move through the lymph chain and engulf and destroy germs. B. bring in cells into the lymph node [your bump] to stop the germs from going anywhere else in the body. C. bring all kind of good cells to your throat so that they can wall the strep off and keep the germs from getting any food or water. D. help your tonsils get bigger with cells that will bring immune cells into your throat to prevent any other infections.

Answer: A Rationale: Lymph nodes serve as centers for proliferation of immune cells at times of infection. Lymph nodes do not sequester germs or immune cells. Tonsils are part of the lymphoid system but are separate from lymph nodes.

A client has been diagnosed with rheumatoid arthritis and asks the nurse what causes this condition. What is the nurse's best response? A. Your immune system is attacking your own tissues as if they were harmful organisms. B. Your white blood cells are failing to respond to the increased inflammation in your joints. C. This is a condition where chronic inflammation causes the loss of bone and joint tissues. D. Your joint tissues are releasing foreign chemicals, triggering your immune system to attack the joint.

Answer: A Rationale: Many autoimmune diseases, such as Hashimoto thyroiditis and rheumatoid arthritis, are caused by impairment in both B and T lymphocyte functions, resulting in cellular damage due to the immune system not distinguishing "self" from "nonself." There is no loss of bone in rheumatoid arthritis; RA does not result from joint tissues releasing "foreign chemicals"; there is an overresponse by the white cells, not a failure to respond.

A school-age child with a history of asthma has brought a note home from school stating there has been one case of meningitis (Neisseria meningitidis) in the school. Since the mother is a nurse, she is very concerned since she knows the portal of entry of this pathogen is: A. inhalation via the respiratory tract such as through breathing or yawning. B. direct contact with a contaminated object such as a pencil. C. ingestion such as when children share their drink with their friends. D. through a cut or abrasion that may occur on the playground.

Answer: A Rationale: Neisseria meningitidis is one of a number of pathogens that invade the body through the respiratory tract.

After several months on a waiting list, a 44-year-old male received a liver transplant 5 days ago. In the last 36 hours, he has developed a rash beginning on his palms and soles, along with abdominal pain and nausea. It has been determined by his care team that the immune response that is causing his symptoms originates not with his own compromised immune components but with those introduced with his new organ. This man's most likely medical diagnosis is: A. Graft-versus-host-disease (GVHD) B. Acute transplant rejection C. Hyperacute organ rejection D. T-cell mediated graft rejection

Answer: A Rationale: Rash, gastrointestinal involvement, and pernicious activity by donor immune cells are the hallmarks of GVHD. The description does not suggest acute or hyperacute transplant rejection and T-cell mediated rejection is not a diagnosis in and of itself, but rather one of the mechanisms of transplant rejection.

A nurse is changing the wound dressing on a coccyx-region pressure injury of an immobilized client. The existing dressing is saturated with both watery, clear discharge and foul, grayish-colored liquid. Which entry in the client's chart most accurately documents these findings? A. "Large amounts of suppurative and serous exudates noted." B. "Purulent discharge and fibrinous exudates noted on existing dressing." C. "Abscess activity noted to coccyx wound." D. "Plasma proteins and membranous exudates present on existing dressing."

Answer: A Rationale: Serous discharge is clear and low in plasma proteins, while suppurative (or purulent) exudates are a mass of degraded cells. An abscess would be physically contained with no discharge, and the exudate is neither fibrinous nor membranous.

A client presents with sudden, violent diarrhea and vomiting after consuming chicken and potato salad 8 hours prior at the beach on a hot day. The nurse knows which statement best matches the phase of the infectious process of food poisoning? A. Maximum impact of infectious process B. Insidious prodromal phase C. Sudden incubation of active replication D. Subacute prodromal phase

Answer: A Rationale: The lack of prodrome and intense symptom onset typify a fulminant illness. The onset described does not characterize a prodromal phase or incubation.

When explaining the final stages of the inflammatory response to pathogens, the nurse should include which educational topics? PRIMEXAM.COM A. How the body can kill the pathogen by generating toxic oxygen and nitrogen products, producing such things as nitric oxide and hydrogen peroxide B. Margination, which is a process whereby white cells (leukocytes) stick to the endothelium and accumulate along the blood vessel C. The increase in vascular permeability, which lets fluids leak into the extravascular tissues D. The promotion of tissue regeneration, whereby monocytes and macrophages produce potent prostaglandins and leukotrienes.

Answer: A Rationale: The latter stages of phagocytosis result in intracellular killing of pathogens, accomplished by several mechanisms, including toxic oxygen and nitrogen products, lysozymes, proteases, and defensins. The metabolic burst pathways generate toxic oxygen and nitrogen products (e.g., nitric oxide, hydrogen peroxide, and hypochlorous acid). Margination is the early stages of the inflammatory response. Vascular changes occur with inflammation but are prior to the final stage. Macrophages arrive within hours at the inflammation site.

A 2-year-old girl has had repeated ear and upper respiratory infections since she was born. A pediatrician has determined a diagnosis of transient hypogammaglobulinemia of infancy. What is the physiologic origin of the child's recurrent infections? A. The child's immune system is unable to synthesize adequate immunoglobulin on its own. B. The child had a congenital absence of IgG antibodies, which her body is only slowly beginning to produce independently. C. The child was born with IgA and IgM antibodies, suggesting intrauterine infection. D. The child lacks the antigen presenting cells integral to normal B cell antibody production.

Answer: A Rationale: Transient hypogammaglobulinemia of infancy is characterized by inadequate communication between B and T cells. IgG would be inherited through placental blood, and an interuterine infection is neither causative or nor synonymous with transient hypogammaglobulinemia of infancy. The diagnosis does not include a lack of antigen presenting cells.

A newly diagnosed HIV-positive adolescent has blood work drawn that includes a CD8+ T-cell count. The nurse knows that which functions of CD8+ T cells listed below will assist the adolescent's immune system in fighting off the viral attack? Select all that apply. A. Release of destructive enzymes B. Trigger intracellular programmed death C. Cause allergens to surround the virus D. Booster antigen-antibody response E. Removal of foreign material from lymph before it enters the blood

Answer: A, B Rationale: The primary function of cytotoxic T (CD8+) cells is to monitor the activity of all cells in the body and destroy any that threaten the integrity of the body. The CD8+ cells destroy target cells by releasing cytolytic enzymes, toxic cytokines, and pore-forming molecules or by triggering membrane molecules and intercellular apoptosis. Removal of foreign material from lymph before it enters the blood is the function of lymph nodes.

Which clients are at increased risk for developing secondary hypogammaglobulinemia, a secondary humoral immunodeficiency? Select all that apply. A. cleint with burns covering more than 45% of the body B. malnourished child who only drinks cow's milk C. client with a history of anxiety and depression D. child allergic to furs, grass, and trees E. client witha a history of seizures controlled by an antiepileptic

Answer: A, B, E Rationale: There are numerous causes of secondary hypogammaglobulinemia, including malnutrition, burns, gastrointestinal loss, nephrotic syndrome and malignancy and as a side effect of certain medications like antiepileptics, antihypertensives, and glucocorticoids. These conditions can result in increase in immunoglobulin loss and/or a decrease in immunoglobulin production. Allergies to enviromental surroundings and anxiety/depression are not causes of secondary hypogammaglobulinemia.

While volunteering in an HIV clinic in a big city, the nurse notices a new mom and her 6-month old child in the waiting room. Upon assessing the infant for possible HIV infection, the nurse will be assessing for which clinical manifestations of HIV infection? Select all that apply. A. Weighing him to determine if he is gaining 1.5 to 2 pounds/month. B. Observing to see if he can roll over from back to stomach. C. Lack the coordination to play with toys/stuffed animals. D. History of repeated episodes of bacterial pneumonia and ear infections. E. Listlessness and poor eye contact.

Answer: A, C, D, E Rationale: Children differ as to their clinical presentation of HIV infection when compared to adults. Failure to thrive (gain weight/height), CNS abnormalities (listlessness), and developmental delays are the most prominent primary manifestation of HIV infection in children. Therefore, assessing the child's weight is important for management of failure to thrive children. PCP (pneumonia) is a major cause of early mortality for HIV-infected children. Assessing weight and developmental tasks of a 6-month old (can the infant roll over?) are normal growth assessments.

An 80-year-old client has a stage 3 decubitus ulcer on the left ischial tuberosity which has not shown much improvement despite optimal local wound treatment. What other interventions should the nurse recommend to promote wound healing? Select all that apply. A. A review of the client's medications B. Positioning the client in a chair three times a day C. Placing the client on an alternating pressure mattress D. Administering corticosteroids E. Nutritional supplements

Answer: A, C, E Rationale: Actions that will promote wound healing in this case include optimizing the client's nutritional status; reviewing the client's medications for any drugs such as corticosteroids that may impede wound healing (it would, therefore, be incorrect to recommend this treatment); and using an alternating pressure mattress to offload the pressure on the area and promote blood flow. Sitting in a chair would place pressure on the ischial tuberosities so would not be recommended.

Tumor necrosis factor-α and IL-1 are major cytokines that mediate inflammation. If the client is developing a systemic response to an infection, the nurse will likely assess which clinical manifestations? Select all that apply. A. Elevated temperature B. Hypertension C. Tachycardia D. Decrease in urine output E. Anorexia

Answer: A, C, E Rationale: IL-1 and TNF-α are mediators of the acute-phase responses associated with infection or injury. Features of these systemic responses include fever [elevated temperature], hypotension, tachycardia [increased heart rate], anorexia, increase in neutrophil count, and increased levels of corticosteroid hormones.

A parent brings a 12-year-old to the emergency room with multiple bite wounds to the arms and hands from a stray cat. Which interventions should the nurse be prepared to apply? Select all that apply. A. A review of the client's immunization record B. Administration of corticosteroids C. Rabies prophylaxis D. Suturing of the wounds E. Irrigation of the wounds F. Administration of prophylactic antibiotics

Answer: A, C, E, F Rationale: Animal bites, and particularly cat bites, are often infected at the time of presentation and require irrigation, debridement, and prophylactic antibiotics; neither suturing nor corticosteroids are recommended. Postexposure rabies prophylaxis should be administered since this was a stray cat and its vaccination history is unknown. The nurse should also inquire as to the client's last tetanus immunization.

Which client is most likely to be a candidate for a thymus transplant as the treatment of choice to reconstitute T-cell immunity? A. A 12-year-old girl with a history of epilepsy and low IgG levels secondary to phenytoin use B. A 7-year-old boy with diagnosis of thymic aplasia whose blood work indicates absence of T cells C. A 6-year-old boy whose pre-B cells are incapable of translation to normal B cells D. A 9-year-old girl who has developed secondary cell-mediated immunodeficiency following a viral infection

Answer: B Rationale: A primary cell-mediated immunodeficiency like true thymic aplasia and congenitally absent T cells usually respond to thymic tissue transplantation and MHC-compatible bone marrow. Low IgG levels secondary to phenytoin use, X-linked hypogammaglobulinemia and selective IgA deficiency are unlikely to be treated with the T cell-focused treatments like thymic tissue transplantation and MHCcompatible bone marrow. Secondary cell-mediated immunodeficiencies are more prevalent than primary deficiency and are frequently associated with viral infections.

A client who is recovering from burn injuries is discussing his prognosis with a physician. Which teaching point about expectations for healing should the physician offer? A. "Once your healing is complete, your skin will be just as strong as before your accident." B. "You may find that the scar is a bit smaller than the area of the wound." C. "You'll find that your new tissue is more elastic and fragile than the rest of your skin D. "The final remodeling phase of healing may last up to 3 months in your case."

Answer: B Rationale: An injury that heals by secondary intention [like burns] undergoes wound contraction. As a result, the scars are often smaller than the original area of the wound. There is nearly always an accompanying loss of strength and elasticity, and remodeling can take over 6 months.

Which aspect of a client's site of inflammation would help the care provider rule out chronic inflammation? A. High levels of macrophages B. Increased neutrophils C. Proliferation of fibroblasts D. Infiltration of lymphocytes

Answer: B Rationale: Chronic inflammation lacks the sudden and marked proliferation of neutrophils that is associated with acute inflammation. Chronic inflammation is associated with increased presence and action of fibroblasts, macrophages, and lymphocytes.

A 24-year-old woman presents with fever and painful, swollen cervical lymph nodes. Her blood work indicates neutrophilia with a shift to the left. She most likely has: A. A mild parasitic infection B. A severe bacterial infection C. A mild viral infection D. A severe fungal infection

Answer: B Rationale: Fever and painful, palpable lymph nodes are nonspecific inflammatory conditions; leukocytosis is also common but is a particular hallmark of bacterial infection. Neutrophilia also indicates a bacterial infection, whereas increased levels of other leukocytes would indicate other etiologies. The shift to the left—the presence of many immature neutrophils—indicates that the infection is severe, because the demand for neutrophils exceeds the supply of mature cells.

While teaching about HIV/AIDS to a group of high school seniors, the school health nurse will begin by explaining the basic facts. Which information will this likely include? A. Like all viruses, HIV is a genetic material made from DNA with long molecules that carry genetic information. B. HIV is different from other viruses since it is a retrovirus that selectively attacks the body's immune cells. C. There are two types of HIV, but the one that is endemic to the U.S. is HIV type 2. D. HIV type 1 for some reason rarely develops into full-blown AIDS.

Answer: B Rationale: HIV is a retrovirus that selectively attacks the CD4+ T lymphocytes, the immune cells responsible for orchestrating and coordinating the immune response to infection. It must change from RNA to DNA through a series of stages in order to get in a cell and begin replication. HIV type 2 is endemic in West Africa but is rarely seen in other parts of the world. People with HIV-2 tend not to develop AIDS.

A student states, "It seems like helper T cells do a lot more than just 'help' the cellular immunity process". Which response listed below best conveys an aspect of the role of CD4+ helper T cells in immunity? A. "Without helper T cells, no antigens would be presented." B. "Helper T cells play a major role in stimulating and regulating the whole process." C. "Without helper T cells, the wrong antibodies would end up being produced." D. "Helper T cells are key to the hematopoiesis that produces all the components of the immune system."

Answer: B Rationale: Helper T cells are central to the regulation, proliferation, and stimulation of the immune system. They do not play a central role in antigen presentation or early hematopoiesis, however. Their absence would not result in incorrect antibody production, but rather insufficient or absent immune response.

The nurse knows that which statement listed below relative to a client with malignant melanoma treated with alpha interferon (IFN-α) is accurate? Alpha interferon (IFN-α): A. will kill certain microorganisms that may help spread the cancer. B. plays an important role in the modulation of the inflammatory response. C. helps keep all the blood levels at a higher level. D. controls the migration of leukocytes to their primary site.

Answer: B Rationale: IFNs are cytokines that primarily protect the host against viral infections and play a role in the modulation of the inflammatory response.

When counseling a male client with suspected HIV, the nurse informs him that if the enzyme-linked immunosorbent assay (ELISA) comes back positive, then: A. no further testing is required since this confirms HIV infection. B. a second test known as the Western blot assay will be ordered to confirm positive HIV status. C. he will be sent to an infectious disease physician for a tissue biopsy to confirm infection. D. if the second test, the Western blot, returns negative, he has not developed a case of full-blown AIDS.

Answer: B Rationale: If ELISA positive, then sent for Western blot assay. If Western blot is positive, diagnosis of HIV confirmed. If Western blot is negative, then the person is NOT infected with HIV.

A health care provider has prescribed blood products for a trauma client with a history of selective immunoglobulin A deficiency (SIGAD) who is going into hypovolemic shock. Which blood product is most appropriate for the nurse to infuse? A. platelets from a single donor B. specially washed erythrocytes from normal donor C. fresh frozen plasma from multiple donors D. cryoprecipitate from pooled donors

Answer: B Rationale: In people diangosed with SIGAD, the frequency of circulating anti-IgA antibodies has been reported to be from 20% to 40%, so anaphylaxis and allergic reactions to blood products are of real concern to practitioners. Therefore, only specially washed erythrocytes from normal donors or erythrocytes from Ig-A deficient donors should be used. The other blood products listed will not raise the hemoglobin level in a client who is hemorrhaging (hypovolemic shock).

Which client would be considered to be in the latent period of HIV infection? A. 16-year-old prostitute who has open sores on her labia that drain purulent secretions. B. 33-year-old heroin drug abuser who has numerous enlarged lymph nodes in his axilla and cervical neck region for the past 4 months. C. 45-year-old alcohol abuser who is reporting excessive vomiting of blood that started 2 weeks ago. D. 24-year-old college student who has developed a chronic cough that will not go away, even after taking two courses of antibiotics.

Answer: B Rationale: In the latent period, which can last up to 10 years, the CD4+ count falls gradually to ~200 cells/μL. Some people experience swollen lymph nodes that are chronically swollen for more than 3 months in at least 2 locations, not including the groin. The lymph nodes may be sore or visible externally.

Three days ago, a mother delivered her full-term infant who had been identified as having an in utero infection. The infant is receiving antibiotic and phototherapy, and the mother is breast-feeding. Which types of immunoglobulins could most reasonably be expected to predominate in the infant's immune system? A. IgA, IgM, IgD B. IgG, IgA, IgM C. IgE, IgG, IgD D. IgM, IgD, Igm

Answer: B Rationale: Infants are born with IgG from transfer across the placenta, while IgA is found in colostrum. IgM is indicative of an in utero infection.

A female dental assistant has developed signs and symptoms of a latex sensitivity, and is undergoing allergy testing as well as blood work. Which component of the assistant's blood work would most likely be the focus of her health care provider's analysis? A. Analysis of class II MHC antigens B. Serum IgE immunoassays C. Serum B-lymphocyte levels D. Serum CD8+ levels

Answer: B Rationale: Latex sensitivity can be either a type I or type IV reaction. Though Th1 levels are relevant in a type IV reaction, IgE analysis is the most common component of relevant blood work. MHC and CD8+ levels are unlikely to be considered.

A student arrives at the health clinic anxious and afraid. The student found an enlarged lymph node in the groin area that is extremely painful to touch and "knows" it is cancer. What information should the health care provider relay to this student about lymphadenitis? A. The student is correct. A warning sign of cancer is a lump or bump that should not be there. B. Not all swollen lymph nodes are due to cancer. It could be caused by an infection in the genital region. C. Swollen lymph nodes that are painful to touch are most likely characteristic of neoplasms. D. Increased sexual activity in college is a precurser to development of genital cancers.

Answer: B Rationale: Localized acute and chronic inflammation may lead to a reaction in the lymph nodes that drain the affected area. This response represents a nonspecific response to mediators released from the injured tissue or an immunologic response to a specific antigen. Painful, palable nodes are commonly associated in inflammatory processes, whereas nonpainful lymph nodes are more characteristic of neoplasms. If the student has experienced an increase in sexual activity, he or she should be evaluated for a sexually transmitted or other genital infection.

The nurse knows which component is needed for long-lasting immunity in a client with a diagnosis of sepsis without the causative agent identified? A. Neutrophils B. Lymphocytes C. Colony-stimulating factors D. Natural killer cells

Answer: B Rationale: Lymphocytes provide lifelong immunity and an antigen-specific response to harmful microorganisms. Neutrophils, macrophages, and natural killer cells do not provide this.

A community health nurse is conducting an education session with a group of pregnant women. One of the clients states, "I am not feeling right about getting a flu vaccine while pregnant." What is the nurse's best response? A. "The influenza vaccine is shown to be safe and will reduce the risk for you getting the flu." B. "By getting vaccinated, you reduce the risk of you and your baby getting influenza." C. "The choice is yours, but none of the substances in the vaccine will harm you or your baby." D. "Vaccines are a personal choice and you should decide what is right for you and your baby."

Answer: B Rationale: Maternal vaccination may offer fetal and neonates passive immunity against common infections such as influenza or herpes zoster. The nurse should not state that vaccines are a personal choice as this does not inform the client of the purpose of the vaccine (to protect both the mother and the fetus/neonate from the virus). Simply saying the vaccine is considered safe is not as informative.

Which client would have a very poor response related to tissue regeneration of his or her injured area? A. 21-day-old infant undergoing a diaphragmatic hernia repair B. 54-year-old male who had a massive MI 4 days ago and came to the ED today for treatment. C. 73-year-old female who is undergoing lithotripsy for kidney stones. D. 33-year-old athlete undergoing surgery to repair a torn MCL in the right knee

Answer: B Rationale: Permanent or fixed cells cannot undergo mitotic division. The fixed cells include nerve cells, skeletal muscle cells, and cardiac muscle cells.

The nurse knows which statement best describes a characteristic trait of Rickettsiae related to Rocky Mountain spotted fever? Rickettsiae: A. are eukaryotic. B. have both RNA and DNA. C. have a distinct spiral-shaped morphology. D. are neither gram-negative nor gram-positive.

Answer: B Rationale: Rickettsiae combine characteristics of bacteria and viruses. Rickettsiae are prokaryotic but not spiral shaped. Rickettsiae are able to be classified by Gram stain like other microorganisms.

Following a spider bite she received while camping, a 20-year-old female presented to the emergency department with rash, edema, and fever and was subsequently diagnosed with serum sickness. Which statement best conveys the physiologic rationale for the broad systemic effects of this event? A. The woman is experiencing diffuse tissue necrosis as a consequence of an Arthus reaction. B. Antigen-antibody complexes have been deposited in a variety of locations throughout the body. C. Antibody binding to specific target cell receptors is bringing about a change in cell function. D. Deposited antibodies are activating her complement system.

Answer: B Rationale: Serum sickness is characterized by the deposition of antigen-antibody complexes in blood vessels, joints, heart, and kidney tissue. The deposited complexes activate complement, increase vascular permeability, and recruit phagocytic cells, all of which can promote focal tissue damage and edema. Serum sickness is not synonymous with an Arthus reaction. Antibody binding to specific target cell receptors causing a change in cell function is characteristic of antibodymediated cellular dysfunction. Serum sickness is not associated with the activation of the complement system.

The nurse is performing discharge teaching for a client who has recovered from sepsis following an open cholecystectomy. What should be the nurse's primary focus? A. Dietary changes needed after cholecystectomy B. Signs and symptoms of infection C. Follow-up appointments with health care providers D. Exercises that are permitted postoperatively

Answer: B Rationale: Survivors of sepsis have a high rate of rehospitalization in the first year (most commonly due to infections), so this would take priority over topics such as dietary or activity restrictions. Follow-up appointments are important, but if symptoms should develop prior to these appointments, the nurse needs to stress the importance of seeking medical attention.

A nurse is interviewing a client with fever, myalgia, headache, and lethargy. Which question is most important for the nurse to ask related to identifying the cause of these symptoms and the possible need for quarantining the client? A. Are these symptoms similar to an illness you have had in the past? B. Have you or any family members traveled outside the country recently? C. Did you receive an influenza vaccine this season? D. Have you been exposed to anyone who has not been vaccinated recently?

Answer: B Rationale: The client is presenting with the nonspecific symptoms of viral illness. Due to the global market and the ease of international travel, there is increasing risk for global pandemics. Knowing if the client has recently visited certain countries could help identify the most likely viruses to which the client may have been exposed. Being vaccinated for local influenzas is not a guarantee the client has or does not have this, or another, virus. Being exposed to an unvaccinated person is only relevant if that person is infectious; because many viral illnesses present with similar symptoms, this question is not helpful. Having had a virus in the past reduces the likelihood that the client is ill with that same virus.

The parent of a child with allergies has been told the child will have a blood test done for immunoglobulin measurement. The nurse should tell the parent that which immunoglobulin will be tested? A. IgG B. IgE C. IgA D. IgM

Answer: B Rationale: The immunoglobulin associated with allergic responses is IgE. Blood tests measuring types and levels of IgE can be done to identify allergens. IgG possesses antiviral, antibacterial, and antitoxin properties. The primary function of IgA is in local immunity on mucosal surfaces. IgM is a complement-fixing immunoglobulin and is instrumental in the ultimate lysis of microorganisms.

A 48-year-old man who has been HIV positive for 6 years has just learned that he has been diagnosed with Kaposi sarcoma (KS). Which fact most accurately conveys an aspect of his diagnosis? A. An opportunistic Epstein-Barr virus underlies the man's KS. B. He is likely to have lesions on his skin, mouth, or GI tract. C. Intense pain was probably his first manifestation of KS D. Heterosexual contact most likely underlies his HIV and subsequent KS.

Answer: B Rationale: The lesions of KS can be found on the skin and in the oral cavity, gastrointestinal tract, and the lungs. More than 50% of people with skin lesions also have gastrointestinal lesions. It is linked with a herpes virus and can often be painless, especially in early stages. Men who have sex with men are at a higher risk of developing KS.

An 8-week-old boy has been recently diagnosed with a severe combined immunodeficiency (SCID). His parents have performed a significant amount of research on the Internet and have brought a large amount of material to discuss with their care provider. Which statement best reflects an accurate understanding of their son's health situation? A. "We read that gene therapy could cure our son; we'd like you to look into that option." B. "Our son likely has a deficiency of B-lymphocytes and can't produce antibodies." C. "We feel guilty, because dietary and environmental factors have been shown to contribute to SCID" D. "The antibodies that our son produces are mismatched to the infections that he was born with, and encounters."

Answer: B Rationale: The pathophysiology of SCID involves normal B cells but also a lack of antibody production because of inadequate T cell help. Gene therapy is not yet a realistic treatment option, and the disease has a genetic basis. Antibodies are not incorrect, but rather inadequate in number.

A client is suspected of having an infection, but the microorganism suspected as the cause cannot be cultured. The client asks how the health care provider will identify the organism. What is the nurse's best response as to the methods used? A. The health care provider will administer empiric antimicrobials, and observe your response to treatment. B. The health care provider will introduce cultured, marked antibodies to your blood sample and observe for a reaction with antigens. C. The health care provider will observe for a cytopathic effect on biopsy tissue samples from the your mucosa. D. The health care provider will release purified antigens into your circulation to observe whether you produce the relevant antibodies.

Answer: B Rationale: The process described is direct antigen detection, in which monoclonal antibodies are labeled with a substance that allows microscopic or overt detection when bound to the pathogen or its products that would be present in the client's blood. While empiric treatment may be used to improve outcomes, this will not help definitively diagnose the pathogen responsible. The observation for a cytopathic effect is a type of viral culture, and the antigens are not injected into the client.

Which procedure reduces the potential for infection primarily by addressing the portal of entry? A. Wiping down common areas with buffered bleach on a regular basis B. Wearing gloves when contact with blood or body fluids is anticipated C. Disposing of soiled clothing and bed linens in a dedicated receptacle D. Isolating clients who have antibiotic-resistant infections

Answer: B Rationale: The wearing of gloves specifically blocks the portal of entry to the health care worker through the use of a physical barrier. Bleaching and cleaning as well as disposing of soiled linen eliminate the source of infection by killing microorganisms, and isolating clients similarly addresses a source by minimizing contact with uninfected persons.

As part of her prenatal education, a 29-year-old woman who is pregnant with her first child is receiving teaching from her primary care provider. Which statement by the woman reflects an accurate understanding of HIV transmission? A. "I know my baby is safe from HIV while in the womb, but the delivery will place him or her at real risk." B. "It's discouraging to know that my breast milk can pass on HIV to my baby." C. "I know it's possible, but it's comforting that the chances of my child contracting my HIV are actually very low." D. "I'm relieved to learn that a caesarean delivery will protect my baby from being born HIV positive."

Answer: B Rationale: Transmission from mother to infant is the most common way that children become infected with HIV. HIV may be transmitted from infected women to their offspring in utero, during labor and delivery, or through breast-feeding. Ninety percent of infected children acquired the virus from their mother. The risk of transmission of HIV from mother to infant is approximately 25%, with estimates ranging from 15% to 45% depending on what country they live in.

Which statement is an accurate descriptor of the role of viruses in human infections? A. Viruses have no genetic material of their own. B. Some viruses are capable of transforming normal host cells into malignant cells. C. Viruses are often implicated in cases of transmissible neurodegenerative disease. D. Viruses require stimulation after a latent period before they are able to produce symptoms.

Answer: B Rationale: Viruses that are categorized as oncogenic are able to induce malignancy in host cells. Viruses have limited genetic material (either RNA or DNA) but no virus lacks genetic material. Transmissible neurodegenerative disease is associated with prions. Not all viruses include a latent period.

Health care team members know recently an increased incidence of infections such as West Nile virus and SARS do not match previously established patterns. Which phenomenon constitutes the most significant contributor to the spread of new diseases in the United States? A. Drug resistance by bacterial and protozoal infections B. Increased ease and speed of travel for individuals and groups C. Genetic variation and mutation by microorganisms D. Decreased living standards and public health standards in urban areas

Answer: B Rationale: While drug resistance, lapses in public health, and genetic variation are all contributing factors in incidents of infection, the increase in new and global diseases is primarily attributed to the ease and frequency of individual and group international travel.

Which process would most likely be considered an anomaly during the cellular phase of inflammation? A. Platelet aggregation B. Vasoconstriction C. Migration of phagocytic white cells D. Macrophage activity

Answer: B Rationale: While vasoconstriction is a component of the immediate inflammatory response, the later cellular phase of inflammation is accompanied by vasodilation. Platelet aggregation, migration of phagocytic white cells, and macrophage activity are all associated with the cellular phase.

After years of going to different physicians with vague symptoms, a 55-year-old client with a history of Hodgkin lymphoma has been diagnosed with a secondary immunodeficiency syndrome. The client asks the nurse what this means. The nurse knows that secondary immunodeficiency disorders may occur if which background information is available? Select all that apply. A. May be inherited as a sex-linked trait B. Usually develop later in life C. May be a result of chemotherapy being used to treat a cancer D. Can result from frequent recurring Staphylococcus aureus infections E. Can occur in clients taking corticosteroids daily.

Answer: B, C, E Rationale: Secondary immunodeficiency disorders develop later in life as a result of other pathophysiologic states such as malnutrition, disseminated cancers, infection of the cells of the immune system, and treatment with immunosuppressive drugs, such as chemo. Primary disorders may be congenital or inherited as sex-linked, autosomal dominant or autosomal recessive traits. Humoral (B-Cell) immunodeficiencies are primary associated with recurrent infections like Staphylococcus aureus.

A 4-year-old boy presents with a chronic cough and swollen lymph nodes. His records show that he has been given antibiotics several times in the past year with limited success, most recently for a liver abscess. He also has a recurring fungal skin condition. Which diagnosis is most likely the cause of this child's manifestations? A. Selective IgA deficiency B. A deficiency in IgG2 subclass antibodies C. Chronic granulomatous disease D. Ataxia-telangiectasia

Answer: C PRIMEXAM.COM Rationale: Chronic granulomatous disease, because it affects phagocytic function, increases susceptibility to soft-tissue infections, particularly of the skin, lungs, lymph nodes, and liver. Selective IgA deficiency and deficient IgG2 subclass antibodies can predispose people to infection, but those infections respond readily to antibiotic treatment. Ataxia-telangiectasia can cause skin and liver problems, but its primary manifestations are ataxia and telangiectasia.

A 23-year-old man has received a recent diagnosis of appendicitis. The nurse providing care for the man is explaining that the inflammation of his appendix is playing a role in his body's fight against the underlying infectious process. Which teaching points should the nurse eliminate from client education? A. "Inflammation can help to remove the body tissue cells that have been damaged by infection." B. "Inflammation will start your body on the path to growing new, healthy tissue at the site of infection." C. "Inflammation helps your body to produce the right antibodies to fight the infection." D. "Inflammation ultimately aids in eliminating the initial cause of the cell injury in your appendix."

Answer: C Rationale: Antibody production is not a noted component of the inflammatory response. Removing damaged cells, generating new tissue, and eliminating the cause of cell injury are all documented components of the inflammatory response.

While undergoing a kidney transplant from a non-family member, the client's transplanted kidney has just had the arterial clamps removed. The OR staff notices that the organ is turning purple with no urine output. When explaining to the family why they had to remove the donor kidney, the nurse will anticipate that the surgeon would likely include which statement? A. Obviously, there has been a mismatch during the human leukocyte antigens (HLA) testing. B. The circulating B and T lymphocytes are just doing their job. C. Hyperacute rejection occurs because antibodies against HLA antigens are deposited in vessels, causing necrosis. D. Previous exposure to the HLA antigens is responsible for the high titers of complement-fixing antibodies that cause the rejection.

Answer: C Rationale: Antibody-mediated rejection can be hyperacute, which occurs almost immediately after vascular reperfusion to graft tissue occurs. Performed antibodies against HLA antigens are deposited in the tissue endothelium and microvasculature where they activate the classic complement pathway, causing tissue necrosis and graft injury.

When explaining to parents what is occurring when their child has an acute bronchial asthma attack, the nurse will emphasize that which mediator is primarily responsible for the bronchial constriction? A. Tree pollen B. Mold dust C. Histamine D. T-lymphocyte proliferation

Answer: C Rationale: Asthma response begins within 5 to 30 minutes of exposure to an allergen. It is mediated by mast cell degranulation and the release of preformed and/or enzymatically activated mediators. These mediators include histamine, serotonin, and acetylcholine. Histamine is the most recognized mediator of type I hypersensitivity reactions and ultimately results in bronchial constriction.

A middle school student is scheduled to receive booster immunizations and the father asks the nurse why the booster is necessary. What characteristic of the adaptive immune system listed below would provide the rationale for the nurse's response? A. Some antibodies require a repeat of the primary immune response. B. Some antibodies have a duration measured in months rather than years. C. A secondary response causes a sharp rise in antibody levels. D. Antigen receptors on CD4+ cells require multiple exposures separated by time.

Answer: C Rationale: Booster immunizations take advantage of the increase in antibodies that accompanies a repeat exposure. The primary immune response cannot be repeated, and antibodies survive beyond several months. Antigen receptors on CD4+ cells do not require multiple exposures.

Which client is most likely to have impairments to the wound healing process? A client with: A. chronic obstructive pulmonary disease. B. a diagnosis of multiple sclerosis and consequent impaired mobility. C. poorly controlled blood sugars with small blood vessel disease. D. congenital heart defects and anemia.

Answer: C Rationale: Diabetes mellitus is strongly associated with impaired wound healing. The other noted pathologies are less causative of deficiencies in the healing process.

An 81-year-old female client in a subacute medical unit of a hospital has developed an oral Candida albicans infection. Which phenomena would the client's nurse suspect as a key contributing factor to her infection? A. The moist and temperature-suited oral environment of the client's mouth B. The ability of fungi to remain latent until the host reaches an immunocompromised state C. Antibiotic therapy that eliminated normal bacterial flora D. The airborne communicability of yeast and molds and subsequent inhalation

Answer: C Rationale: Elimination of bacterial flora that normally keeps colonizing fungi in check can induce the proliferation of fungi. The environment of the client's mouth provides a conducive environment, but with an intact immune system is rarely a direct contributor to colonization. Latency is not a common trait of Candida albicans, and airborne transmission is not likely to be a causative factor.

A 63-year-old woman has begun a diet that purports to minimize the quantity and effects of free radicals in her body. Which physiologic process could best underlie her care provider's teaching about her new diet? A. Free radicals act as direct mediators in the inflammatory process. B. Free radicals inhibit the inflammatory response, limiting pre-adaptive response to infection. C. Free radicals increase cytokine expression and adhesion molecule levels, resulting in increased inflammation. D. Free radicals contribute to atherosclerosis and decreased immune response.

Answer: C Rationale: Free radicals are thought to bring about an inappropriate inflammatory response by increasing cytokine and numbers of adhesion molecules. They are not direct mediators of inflammation and are not associated with decreased immune response, but rather inappropriate inflammation. Free radicals are not associated with inhibition of the inflammatory response.

Which infectious agent would be a site-specific pathogen and not spread throughout the body via the circulatory system? A client diagnosed with: A. B. burgdorferi caused by a tick bite. B. Salmonella typhi related to ingestion of contaminated food or water. C. Helicobacter pylori diagnosed after an endoscopic procedure. D. N. meningitidis infection of child in a daycare setting.

Answer: C Rationale: H. pylori is an extreme example of a site-specific pathogen (localized infectious disease). The other three pathogens are called systemic pathogens because they can disseminate throughout the body via circulatory system.

A client diagnosed with H. pylori asks the nurse, "How can an infection occur in the stomach since it is an acid environment?" The nurse responds: A. "We have many infectious agents that can live in an acidic environment with a pH more than 8.0." B. "H. pylori is a virus and is still being researched as to how it is able to survive in the stomach acids." C. "H. pylori produces an enzyme called urease that converts gastric juices into ammonia, which neutralizes the acidic stomach environment." D. "This parasite secretes an enzyme called coagulase, which protects the pathogen from the antibodies."

Answer: C Rationale: H. pylori, the infectious cause of gastritis and gastric ulcers, produces the urease enzyme on the outer cell wall. The urease converts gastric urea into ammonia, thus neutralizing the acidic environment of the stomach and allowing the organism to survive in this hostile environment.

A client who has a diagnosis of an autoimmune disease asks his nurse why it is that their immune system does not attack all of the cells that make up his body. Which of the following aspects of pathogen recognition in the innate immune system listed below would underlie the nurse's response? A. Normal host cells excrete inhibitory proteins that are detected by natural killer cells. B. Intraepithelial lymphocytes and natural killer cells possess specific, highly diverse receptors. C. Pattern recognition receptors (PRRs) ensure cells are correctly identified. D. Leukocytes possess pathogen associated molecular patterns (PAMPs)

Answer: C Rationale: PRRs recognize the structure of invaders and thus prevent activation by healthy somatic cells, though neither intraepithelial lymphocytes nor natural killer cells possess the high level of specification and diversity of receptors associated with the adaptive immune system. Host cells do not excrete inhibitory proteins and PAMPs exist on pathogens, not leukocytes.

The nurse knows that the cells primarily programmed to remove the invading organisms and remember the antigen to respond rapidly during the next exposure are: A. CD4 and CD8 cells. B. Natural Killer (NK) cells and macrophages. C. T-lymphocytes and B-lymphocytes. D. White blood cells and platelets.

Answer: C Rationale: T-lymphocytes and B-lymphocytes are unique in that they are the only cells in the body capable of recognizing specific antigens present on the surfaces of microbial agents and other pathogens.

A family of five vacationed together, but only the client contracted traveler's diarrhea. The client asks the nurse, "We all ate and drank the same things. Why am I the only one with this condition?" What should the nurse offer to the client as the most likely reason for susceptibility for this condition? A. You are male and your family members are all female. B. You are taking a beta blocker for hypertension. C. You are taking a proton pump inhibitor for gastric reflux disease. D. You are the only obese member of the family.

Answer: C Rationale: Taking a proton pump inhibitor will reduce gastric acidity, which makes the client more susceptible to infection by the oral route because the number of ingested microorganisms surviving the gastric environment is greater. Risk factors for traveler's diarrhea are not related to hypertension, weight, or gender.

A nurse is providing care for a 17-year-old boy who has experienced recurrent sinus and chest infections throughout his life and presently has enlarged tonsils and lymph nodes. Blood work indicated normal levels of B cells and free immunoglobins but a lack of differentiation into normal plasma cells. The boy is currently receiving intravenous immunoglobulin (IVIg) therapy. What is the boy's most likely diagnosis? A. X-linked hypogammaglobulinemia B. Transient hypoglobulinemia C. Common variable immunodeficiency D. IgG subclass deficiency

Answer: C Rationale: The lack of a terminal differentiation from B cells into plasma cells is the hallmark of common variable immunodeficiency. Recurrent infections, enlarged lymph nodes and tonsils, and IVIg therapy are also commonly associated.

Staphylococcus aureus commonly found in the skin, nares, and other body sites of clients without any signs and symptoms of infection is known as which condition? A. An opportunistic infection B. A parasitic infestation C. Bacterial colonization D. A saprophytic infection

Answer: C Rationale: The ongoing presence in the body of bacteria that do not cause infection is called colonization. Opportunistic infection by ordinarily nonpathogenic organisms can occur in immunosuppressed hosts. Parasitism is a condition in which the organism exists at the expense of, and without benefiting, the host. Saprophytes do not cause infection.

The spirochete Leptospira is primarily transmitted to farmers by: A. an airborne mechanism. B. exposure to spores in the environment. C. direct contact with infected animals. D. a mosquito bite.

Answer: C Rationale: The pathogenic leptospires infect a wide variety of wild and domestic animals. Infected animals release the organisms into the environment through the urinary tract. Transmission to humans occurs by contact with infected animals or urine-contaminated surroundings. This spirochete is not transmitted by airborne, spores, or a fomite.

A client has been inhaling viruses periodically while on a cross-country flight. Which situation listed below would most likely result in the stimulation of the client's T lymphocytes and adaptive immune system? A. Presentation of a foreign antigen by a familiar immunoglobulin B. Recognition of a foreign major histocompatibility complex (MHC) molecule C. Recognition of a foreign peptide bound to a self-major histocompatibility complex (MHC) molecule D. Cytokine stimulation of a T lymphocyte with macrophage or dendritic cell mediation.

Answer: C Rationale: The stimulation of T cells requires the recognition of a foreign peptide bound to a self-major histocompatibility complex (MHC) molecule. Immunoglobulins do not play an antigen-presenting role, and foreign MHC molecules and cytokines do not stimulate the adaptive immune system.

A potential donor is angry at the personal nature of the questions about HIV risk factors that he is required to answer at a blood collection center, and states that simple blood testing should suffice. How can the nurse at the center best respond? A. "There are some very uncommon subtypes of the HIV virus that are not detectable by current testing methods." B. "There's a chance that persons who are asymptomatic but HIV positive can have their antibodies missed by serum testing." C. "There's a period shortly after someone is infected with HIV when blood tests might still be negative." D. "Even though blood tests are completely accurate, the high stakes of blood donation and transfusion mean that double measures are appropriate."

Answer: C Rationale: The time after infection and before seroconversion is known as the window period, during which HIV antibody screening may be negative. Potential donors are thus screened to identify potential risk factors. Undetectable subtypes of HIV do not exist and individuals who are asymptomatic are still able to be accurately tested.

Which phenomenon would be least likely to result in activation of the complement system? A. Recognition of an antibody bound to the surface of a microbe B. Increase tissue blood flow and capillary permeability so fluids/proteins can leak into the area. C. Activation of toll-like receptors (TLRs) on complement proteins. D. Direct recognition of microbial proteins.

Answer: C Rationale: Toll-like receptors are not associated with the complement system. The complement system may be activated by antibody recognition, mannose-binding or microbial protein recognition

The physician knows the client, G1P0, has correctly understood the prenatal education regarding sexually transmitted infection as evidenced by which statement listed below? A. "Out of the various infectious agents, gonorrhea and chlamydia pose the greatest risks of transmission from mother to child." B. "I know that my baby will need observation for HIV signs and symptoms in the weeks following my delivery." C. "My baby could become infected either across the placenta or during the birth itself." D. "If I receive prophylactic immunization I will reduce my baby's chance of being born with an illness."

Answer: C Rationale: Vertical transmission may occur across the placenta in utero or during the birth event itself. Cytomegalovirus and HIV are the most common congenital infections; HIV signs and symptoms are not immediately apparent in the infant. Prophylactic immunization is not noted as a proven intervention for preventing vertical transmission.

A 39-year-old female with HIV has been characterized as a typical progressor by her care team, and is experiencing an increase in her manifestations and complaints as her CD4+ count declines. Which health problem would her care team most likely attribute to a cause other than her HIV? A. Her recent diagnosis of bacterial pneumonia B. Her esophagitis that has been linked to herpes simplex infection C. Her decreased bone density and recent fractures D. Her increasing confusion and disorientation

Answer: C Rationale: While pneumonia, esophagitis, and cognitive deficits are all well documented manifestations of HIV, changes in bone density are less likely to be a direct result of the virus.

A client with a large decubitus ulcer asks the nurse, "How it is possible for a wound this deep to ever fully heal?" What is the nurse's best response? A. Deep wounds like this require surgical intervention and skin grafting before they will be healed. B. It will be difficult, but you have many experts planning your care so the chances of healing are good. C. Once the first layer of new granulation tissue is formed, the speed of the repair will increase greatly. D. With adequate resources, the body is capable of regenerating tissue and blood flow to the area over time.

Answer: D PRIMEXAM.COM Rationale: Even large wounds can heal over time given adequate blood and nutritional supply. The formation of early granulation tissue is one of the more rapid phases; the subsequent healing by secondary intention takes much longer. Surgery may be required, but this is not true for all large wounds. Telling the client there are experts planning the care does not address the client's question and is dismissive.

Which type of pneumonia is best characterized by an infective agent that produces sputum samples with a peptidoglycan cell wall, expresses endotoxins, replicates readily in broth and on agar, grows in clusters, has pili, and does not stain when exposed to crystal violet? A. Chlamydial B. Viral C. Mycoplasmal D. Bacterial

Answer: D Rationale: Although chlamydiae, viruses, and mycoplasmas all can cause pneumonia, only bacteria have all of these characteristics. Chlamydiae and viruses are obligate intracellular organisms and therefore would grow only in cell culture, and mycoplasmas lack the peptidoglycan cell wall typical of bacteria.

A 72-year-old client being treated for rheumatoid arthritis is admitted with suspected ehrlichiosis. What should the nurse implement as a treatment priority? A. Isotonic intravenous fluids at a minimum rate of 200 mL/hr B. Collection of appropriate blood and fluid cultures C. Isolation of the client using contact precautions D. Administration of empirically chosen antibiotics

Answer: D Rationale: Because the client is older and has rheumatoid arthritis, the nurse should expect a state of immunosuppression. This can be related to the chronic illness and age alone, but may be made worse if the client is on immunosuppressive therapy such as methotrexate or prednisone, which are typical treatments for the client with rheumatoid arthritis. This creates a high risk for the development of sepsis secondary to ehrlichiosis. Evidence validates the importance of empirical antibiotic treatment when the nurse suspects ehrlichiosis, because a fulminant and lifethreatening infection is likely with immunocompromised clients. Only if the client's blood pressure has dropped would intravenous therapy be indicated. Empirical treatment should not be delayed for specimen collection. Routine hospital blood cultures cannot detect Ehrlichia. Laboratory testing for the bacterium involves polymerase chain reaction (PCR) assay.

The nurse knows a drug in a category identified as a colony-stimulating factor (CSF) helps: A. cells engulf and digest microbes that want to attach to cell membranes and destroy normal cell function. B. produce cells that will be the first responder cells to protect against cancer formation. C. stimulate the person's immune system so that they can kill their own cancer cells. D. to stimulate bone marrow to produce large numbers of mature cells such as platelets and erythrocytes.

Answer: D Rationale: CSFs participate in hematopoiesis by stimulating bone marrow pluripotent stem and progenitor or precursor cells to produce large numbers of mature platelets, erythrocytes, lymphocytes, neutrophils, and monocytes.

The nurse in the emergency department knows clients exposed to Clostridium botulinum, an agent of bioterrorism, would likely be exhibiting which clinical manifestation listed below? A. Blindness and respiratory distress B. Hemorrhage from all orifices resulting in signs of shock and coma C. Frothy, odiferous diarrhea and dehydration D. Muscle weakness in extremities eventually leading to paralysis of respiratory muscles

Answer: D Rationale: Clostridium botulinum causes neuromuscular paralysis and is listed as a category A agent.

A 1-year-old child who has experienced low platelet counts and bacterial susceptibility has been admitted to a pediatric medical unit of a hospital for treatment of Wiskott-Aldrich syndrome. The nurse who has admitted the child to the unit would anticipate which short-term and longer-term treatment plans? A. Transfusion of clotting factors XII and XIII and serum albumin; splenectomy B. Neutropenic precautions; fresh frozen plasma transfusions; treatment of gastrointestinal symptoms. C. Intravenous immunoglobulin (IVIg) treatment; thyroidectomy D. Treatment of eczema; management of bleeding; bone marrow transplant

Answer: D Rationale: Common interventions for Wiskott-Aldrich syndrome involve controlling eczema, managing bleeding due to low platelets, and ultimately bone marrow transplant. The other noted interventions are not associated with the treatment of Wiskott-Aldrich syndrome

When educating a client with a wound that is not healing, the nurse should stress which dietary modifications to ward off some of the negative manifestations that can occur with inflammation? A. Increase the amount of calcium in the diet, especially drinking milk and eating cheese. B. This is the one time whereby you should eat more fat (both polyunsaturated and saturated) so you can absorb more fat-soluble vitamins. C. Since there is a loss of plasma proteins, you should increase your intake of organ meats like liver. D. Increase your intake of oily fish and fish oil so that you will increase absorption of omega-3 polyunsaturated fatty acids.

Answer: D Rationale: Dietary modification of the inflammatory response through the use of omega-3 polyunsaturated fatty acids, specifically eicosatetraenoic acid and docosahexaenoic acid, which are present in oily fish and fish oil, may be effective in preventing some negative manifestations of inflammation.

A client with a surgical wound has developed excessive granulation tissue extending above the wound edges. Which action should the nurse take? A. Contact the health care provider to order wet-to-dry dressings to promote debridement of the tissue. B. Take no action; this overgrowth of tissue is referred to as "keloids" and is not amenable to treatment. C. Apply an occlusive dressing to increase moisture in the wound bed as excessive granulation tissue is due to dryness. D. Ask the health care provider about surgical options to remove the excess tissue and promote wound healing.

Answer: D Rationale: During the proliferative phase of healing, excessive granulation tissue, sometimes referred to as "proud flesh," may form and extend above the edges of the wound, preventing reepithelialization. The nurse should inquire about surgical removal or chemical cauterization to allow healing to proceed. This is not necrotic tissue and would not be responsive to debridement via wet-to-dry dressings. This tissue differs from keloids, which is an overgrowth of scar tissue (rather than granulation tissue) and is not caused by a dry wound bed.

A hospital client has a large, superficial wound on her elbow that was the result of shearing action when she was moved up in her bed. The client's husband mentions that the wound looks infected and irritated since it is completely red. Which response by the nurse would be inappropriate? A. "Even though it is red, it doesn't mean that the wound is infected." B. "The red areas show that there is enough circulation to facilitate healing." C. "Those are fresh blood vessels that are a sign of healthy healing." D. "A thin sheet of blood clotting is actually desirable and not a sign that your wife's wound is infected."

Answer: D Rationale: Granulation tissue indicates sufficient circulation and angiogenesis associated with healthy wound healing. Granulation tissue consists of new blood vessels, not clotted cellular components.

A 19 year-old intravenous drug user was exposed to the HIV virus 3 weeks ago and is experiencing a rapid proliferation in viral load. Which statement best captures an aspect of the process of HIV replication that underlies this proliferation? A. Free HIV RNA is able to attach to the cell coat of CD4+ cells. B. The cytoplasm of CD4+ cells provides a protected environment for the replication of RNA by HIV. C. Expression of reverse transcriptase by CD4+ cells allows replication of HIV cells rather than new lymphocytes. D. HIV is able to change its RNA into DNA to allow for replication by CD4+ cells

Answer: D Rationale: In order for the HIV to reproduce, it must change its RNA into DNA. It does this by using the reverse transcriptase enzyme. Reverse transcriptase makes a copy of the viral RNA, and then in reverse makes another mirror-image copy. The result is double-stranded DNA that carries instructions for viral replication. HIV RNA does not directly attach to CD4+ cells and RNA is not replicated by HIV itself in the CD4+ cytoplasm. Reverse transcriptase is not produced by CD4+ cells, and CD4+ cells do not directly produce new lymphocytes.

A client presented to the emergency department of the hospital with a swollen, reddened, painful leg wound and has been diagnosed with methicillin-resistant Staphylococcus aureus (MRSA) cellulitis. The client's physician has ordered a complete blood count and white cell differential. Which blood component would the physician most likely anticipate to be elevated? A. Basophils B. Eosinophils C. Platelets D. Neutrophils

Answer: D Rationale: Increased neutrophils are associated with inflammation, in general, and bacterial infections in particular. Platelets play a role in inflammation but their levels would not rise to the same extent as would neutrophils. Eosinophils are not strongly associated with bacterial infection and basophils would not increase to the same degree as neutrophils.

A 79-year-old female resident of an assisted living facility receives care from a community nurse on a regular basis for treatment of a chronic venous leg ulcer. Which factor would the nurse be most justified in ruling out as a contributing factor to the client's impaired wound healing? A. A lower skin collagen content than in younger adults B. Decreased fibroblast synthesis C. Slow re-epithelialization D. Decreased antibody levels

Answer: D Rationale: Older adults do not normally have diminished antibody levels. Low collagen levels, decreased fibroblast activity, and slow re-epithelialization are common impediments to wound healing in older adults.

Which situation can best be characterized as an example of passive immunity? A. A 6-month-old infant receives his scheduled immunization against measles, mumps, and rubella. B. A 9-year-old boy is immune to chickenpox after enduring the infection 1 year previous. C. An 8-year-old girl recovers from a respiratory infection after intravenous antibiotic treatment. D. A 6-week-old infant receives antibodies from his mother's breast milk.

Answer: D Rationale: Passive immunity involves the transfer of antibodies from an outside source, such as those from breast milk. Immunization and recovery from illness involve active immunity.

The nurse knows which of statement below is appropriate to be included in an education session for a 21-year-old male with a diagnosis of malaria? A. "Your infection likely began with the introduction of fertilized protozoal ova from a mosquito." B. "The protozoa responsible have hijacked the genetic material of your cells in order to reproduce." C. "You are very tired because the pathogens are utilizing the ATP that your own cells need." D. "The infectious organisms are considered tiny, single-celled animals, given their complete eukaryotic machinery."

Answer: D Rationale: Protozoa possess full eukaryotic machinery, including organelles and a nucleus. Ova are associated with helminths. Protozoa do not utilize the host's genetic material for reproduction. Appropriation of ATP is associated with chlamydiae.

A 40-year-old woman who experiences severe seasonal allergies has been referred by her family physician to an allergist for weekly allergy injections. The woman is confused as to why repeated exposure to substances that set off her allergies would ultimately benefit her. Which phenomenon best captures the rationale for allergy desensitization therapy? A. Repeated exposure to offending allergens binds the basophils and mast cells that mediate the allergic response. B. Allergens in large, regular quantities overwhelm the IgE antibodies that mediate the allergic response. C. Repeated exposure stimulates adrenal production of epinephrine, mitigating the allergic response. D. Injections of allergens simulate production of IgG, combining with the antigens to prevent activation of IgE antibodies.

Answer: D Rationale: Repeated exposure to allergens causes an increase in IgG, which binds with antigens before they can stimulate IgE. It does not bind mast cells or basophils, nor does it overwhelm the IgE antibodies or stimulate epinephrine production.

A client has been identified as having an excess of macrophage inhibitory factor, causing the client to have inhibited movement and activity of macrophages. Which process listed below would the health care team member expect to remain unaffected? A. Amplification of the immune response B. Destruction of virus-infected or tumor cells C. Initiation of adaptive immunity D. Specificity and memory of the immune response

Answer: D Rationale: Specificity and memory are the defining characteristics of the adaptive immune system, and macrophages do not perform this particular role. Amplification of the immune response, destruction of virus-infected or tumor cells and initiation of adaptive immunity are all components of macrophage activity

A 60-year-old male client with an acute viral infection is receiving interferon therapy. The nurse is teaching the client about the ways interferon differs from other anti-infective therapies. What point should the nurse include? A. "Interferon will only target the cells affected by the virus and not affect other cells." B. "Interferon helps limit macrophage activity to reduce the reaction to the infection." C. "Interferon will suppress apoptosis to prevent the death of healthy cells." D. "Interferon stimulates the activity of natural killer cells that attack viruses."

Answer: D Rationale: The interferons (IFNs) are cytokines that enhance the cellular immune response to viruses. IFNs activate NK cells and also activate macrophages rather than suppress them. They also promote apoptosis rather than suppress it. Interferons not only target affected cells but also induce an antiviral state in uninfected cells.

A 53-year-old female hospital client has received a kidney transplant following renal failure secondary to hypertension. The teaching prior to transplant made her aware that she would need to take anti-rejection drugs for the rest of her life. Which aspect of the immune system underlies this necessity? A. The lack of identifiable major histocompatibility complex (MHC) molecules will stimulate the innate immune response. B. Donor organ antibodies will be identified as foreign and stimulate an immune response. C. Anti-rejection drugs will stimulate the production of familiar MHC molecules. D. MHC molecules will never develop in the cells of the donor organ and effector cells will be continually stimulated.

Answer: D Rationale: The lack of familiar MHC molecules will stimulate an immune response by effector cells in the absence of anti-rejection drugs. An innate immune response is not central to the response, but rather the adaptive immune system. Lack of known MHC molecules, not foreign antibodies, accounts for the immune response, and familiar MHC molecules will not be produced by the donor kidney cells.

A person who has been diagnosed with HIV infection 12 years ago and still has a CD4+ cell count of 800 cells/μL and a low viral load is considered to be in which clinical group? A. Rapid progressor B. Typical progressor C. Delayed progressor D. Long-term nonprogressor

Answer: D Rationale: There is a subset of slow progressor, called the long-term nonprogressor, who account for 1% of all HIV infections. These people have been infected for at least 8 years, are antiretroviral naïve, have high CD4+ cell counts, and usually have very low viral loads. They are being investigated to determine how they maintain viral suppression of HIV.

A client has been diagnosed with herpes simplex virus. The client states that, "modern medicine produces more and more antivirals every year and so the treatment should be simple." Which statement is the best response? A. "The recent rise of drug resistance has significantly hampered the elimination of viruses." B. "The cell coat of viruses is particularly resilient to the available synthetic antivirals." C. "The use of antivirals is severely limited by the unwanted adverse effects that they cause." D. "Treatment options for viruses are often limited because what destroys viruses often damages your own body cells."

Answer: D Rationale: Viruses are difficult to treat because interference with their replication often requires interference with the body's cell replication processes. Though they do exist with antiviral treatments, drug resistance and side effects are phenomena more closely associated with antibacterials. Antivirals act upon DNA or RNA synthesis, not the cell wall.

the incidence of people who are at risk for developing hantavirus while staying in Yosemite National Park.

As of Nov. 1, 2012, there were a total of 10 confirmed cases of hantavirus infection in people who were recent visitors (mid-June to end of August, 2012) to Yosemite National Park. Three visitors with confirmed cases died. Health officials believe that 9 out of the 10 cases of hantavirus were exposed while staying in Curry Village in the Signature Tent Cabins. This is an example of: -the low rate of morbidity one can expect while traveling to Yosemite National Park -the incidence of people who are at risk for developing hantavirus while staying in Yosemite National Park. -the prevalence of hantavirus one can anticipate if he or she is going to vacation in Yosemite National Park. -what the anticipated mortality rate would be if a family of five were planning to vacation in Yosemite National Park

primary prevention

As part of a community class, student nurses are developing curriculum to teach expectant parents the importance of having their child properly secured in a child safety seat. During the class, the students are going to have a safety officer examine the car seats that the parents have installed in their vehicle. This is an example of which type of prevention? •Prognosis enhancement •Secondary prevention •Tertiary prevention • Primary prevention

"The greater the skin resistance, the greater the amount of deep and systemic damage a victim is likely to incur."

As part of a first aid class, a health care instructor is teaching a group of industrial workers about how electrical injuries can cause cell damage. Which statement made by one of the workers indicates that further teaching is necessary? -"The greater the skin resistance, the greater the amount of deep and systemic damage a victim is likely to incur." -"Resistance to flow is the phenomenon that transforms electrical energy into heat." -"The particular pathway that a current takes through the body is very significant." -"The most severe damage is likely to occur where the current enters and leaves the body."

All of the men who had high PSA levels developed prostate cancer: several men who had low PSA levels also developed prostate cancer.

As part of a screening program for prostate cancer, men at a senior citizens center are having their blood levels of prostate-specific antigen (PSA) measured. Which statement would best characterize high positive predictive value but low negative predictive value for this screening test? -Men who had low PSA levels also displayed false positive results for prostate cancer; men with high levels were often falsely diagnosed with prostate cancer -All of the men who had low PSA levels were cancer-free; severa men who had high levels also remained free of prostate cancer. -All of the men who had high PSA levels developed prostate cancer: several men who had low PSA levels also developed prostate cancer. -The test displayed low sensitivity but high specificity.

They tend to have a more uniform symptomology than autosomal dominant disorders.

As part of an orientation to a genetic counseling practice, a group of medical students are differentiating between autosomal recessive disorders and autosomal dominant disorders. Which statement is true of autosomal recessive disorders? -They tend to have a more uniform symptomology than autosomal dominant disorders. -The associated disorders are usually attributable to abnormalities in structural proteins. -They can manifest when present in one or both gene pairs. -There is a 1 in 2 chance of an affected child in each pregnancy with an affected mother.

"Your developing baby is most vulnerable during the first 2 months of your pregnancy."

As part of her prenatal care, a pregnant woman and her partner are being taught by a community health nurse. Which point about the teratogenic effects of different substances should the nurse include in teaching? -"Your developing baby is most vulnerable during the first 2 months of your pregnancy." -Your best option is to avoid using any drugs during your pregnancy" -Keep in mind that a high percentage of genetic abnormalities are attributable to drug origins.! -"You need to be very careful with vitamin D and its derivatives

"Ferritin is a stored form of iron that indirectly shows me whether you would benefit from iron pills."

As part of his diagnostic workup, a 77-year-old male is having his ferritin level analyzed. Which explanation by the health care provider regarding the significance and rationale for this test is most accurate? "Ferritin is a stored form of iron that indirectly shows me whether you would benefit from iron pills." "Ferritin is a protein-iron complex that allows your red blood cells to make use of the iron that you consume in your diet." "Ferritin is the form of iron that is transported in your blood plasma to red blood cells that need it." "Ferritin is the activated and usable form of iron that your red blood cells can use to transport oxygen."

The nurse is caring for a client hospitalized yesterday with acute coronary syndrome. The client also has difficulty dealing with life stressors. On what should the nurse focus as the priority of care?

Assessing the heart rate and blood pressure and interviewing the client about chest pain

The clinician suspects a spinal cord injury client is developing autonomic dysreflexia. Which assessment findings would confirm the development of this complication? Select all that apply. Skin covered with macular rash BP 180/98 Complains of a pounding headache Pulse rate 49 Cold, cyanotic lower legs.

BP 180/98 • Complains of a pounding headache Pulse rate 49

U T lymphocytes E Macrophages B lymphocyte cells Natural killer (NK) cells

Blood-borne cancerous cells have recently spread from a woman's primary pancreas tumor to her bones. Which components of the woman's immune system are likely to be directly involved in the attempt to eradicate the potential metastasis? Select all that apply. T lymphocytes Mast cells E Macrophages B lymphocyte cells Natural killer (NK) cells

When explaining how carbon dioxide combines with water to form carbonic acid as part of acid-base lecture, the faculty instructor emphasizes that which enzyme is needed as a catalyst for this reaction?

Carbonic anhydrase

Which client, when faced with acute stressful situations, would be considered highest risk for becoming noncompliant with his/her medication regimen?

Client with end-stage renal failure experiencing electrolyte imbalances related to having trouble sticking to a prescribed diet.

"Many times, kidney cancer goes completely undetected until a metastatic lesion is found in the lung."

During a discussion with a client recently diagnosed with stage IV kidney cancer, which statement is most accurate? -Most people ignore the blood they see in their urine thinking they have a kidney stone." -"You likely had some kidney pain that you associated with a back strain instead of cancer. -"The health care community needs to do a better job informing people that blood clots in the urine is not normal. -"Many times, kidney cancer goes completely undetected until a metastatic lesion is found in the lung."

"A type of paralysis that affects movement on both sides of the body. It may even involve the respiratory muscles."

During a flu shot clinic, one of the questions asked relates to whether the client has a history of Guillain-Barré syndrome. The client asks, "What is that?" How should the clinician reply? A degenerative discase where you have trouble walking without the help of a can or walker.. "A type of paralysis that affects movement on both sides of the body. It may even involve the respiratory muscles." "Influenza-like illness where you had fever and chills for 2 to 3 days after your last flu shot." "Swelling of your arm where you got your flu shot and maybe your eyes and lips had some swelling as well

Treatment needs to be sought immediately so that the buildup of lactic acid is limited and cellular changes can be reversed.

During a myocardial infarction (MI), a client with a 97% occlusion of the left descending artery develops ventricular dysrhythmias due to the amount of ischemia occurring in the myocardium. While providing education about MI's, which statement is most accurate to share with this client? -Permanent damage will occur in the myocardium if the vessel is not opened within a 1-2 minute window following the occlusion. -The body will grow new genes thru the process of angiogenesis, thereby avoiding any permanent damage to the myocardium. -Once the oxygen supply has been occluded, cellular changes are irreversible even if oxygenation is restored. -Treatment needs to be sought immediately so that the buildup of lactic acid is limited and cellular changes can be reversed.

A client is brought to the ED with reports of SOB. Assessment reveals a full, bounding pulse, severe edema, and audible crackles in lower lung fields bilaterally. The nurse notifies the physician to obtain orders for which of these problems?

Fluid volume excess

The nurse is caring for a client with left-sided heart failure causing chronic activation of the renin-angiotensin-aldosterone system (RAAS). What is the nurse's priority assessment?

Fluid volume excess

The well-differentiated, neoplastic cells are clustered together in a single mass.

Following a biopsy, a 54-year-old man has been diagnosed as having a benign neoplastic tumor. Which characteristic most likely applies to his tumor? -The tumor may secrete hormones, cytokines -The well-differentiated, neoplastic cells are clustered together in a single mass. -The tumor is poorly approximated and has the potential to break loose. -It has a rapid rate of growth and can induce ischemia

BP 90/60, heart rate 132, excess bleeding, and hematoma noted at insertion site.

Following a bone marrow biopsy, which assessment would indicate the client is experiencing a complication as a result of this diagnostic procedure? Respiratory rate 24, complaining of pain at insertion site. BP 130/80, oxygen saturation 95%, crackles heard on inspiration. BP 90/60, heart rate 132, excess bleeding, and hematoma noted at insertion site. Heart rate regular 64 bpm, temperature 99.6°F (37.5°C) orally

Blood has accumulated between the man's dura and subarachnoid space.

Following a collision while mountain biking, the diagnostic work up of a 22-year-old male has indicated the presence of an acute subdural hematoma. Which pathophysiologic process most likely underlies his diagnosis? A traumatic lesion in the frontal or temporal lobe has resulted in increased ICP. Blood has accumulated between the man's dura and subarachnoid space. Vessels have burst between the client's skull and his dura Blood has displaced CS in the ventricles as a consequence of his coup-contrecoup injur.

increased thrombopoietin levels

Following a course of measles, a 5-year-old girl developed scattered bruising over numerous body surfaces and was diagnosed with immune thrombocytopenic purpura (ITP). As part of her diagnostic workup, blood work was performed. Which result is most likely to be considered unexpected by the health care team? Increased thrombopoietin levels Normal leukocyte levels Normal vitamin K levels Decreased platelet count

"Though she still goes through a cycle of sleeping and waking, her condition is unlikely to change."

Following a motor vehicle accident 3 months prior, a 20-year-old female who has been in a coma since her accident has now had her condition declared a persistent vegetative state. How can her care providers most accurately explain an aspect of her situation to her parents? *'Your daughter has lost all her cognitive functions as well as all her basic reflexes." "Your daughter's condition is an unfortunate combination with total loss of consciousness but continuation of all other normal brain function." "Though she still goes through a cycle of sleeping and waking, her condition is unlikely to change." 'If you on the care team notices any spontaneous eye opening. then we will change our treatment plan

neutrophils

Following an injury resulting in a small cut from a knife, the first cells to go to the area of the cut would be the: Basophils Neutrophils Albumin Ervthrocytes

Weakness or slight paralysis affecting one side of the body Periods of unconsciousness aphasia at times

Following head injury from a fall, the child's CT scan reveals a moderate brain injury contusion. Which manifestations will the nurse more than likely assess on this child that supports this diagnosis? Select all that apply. Weakness or slight paralysis affecting one side of the body Periods of unconsciousness 17 Nuchal rigidity Coma with total paralysis Aphasia at times

A 14 yr old boy, appearing to be intoxicated, arrives in the emergency dept where the EMTs report the boy denies consuming anything out of the ordinary. However, an open antifreeze container was found in the boy's room. Which intervention is likely to be prescribed to treat the client's symptoms?

Fomepizole

Client with chronic obstructive pulmonary disease (COPD) who is diagnosed with pneumonia, most likely from a virus

From the assigned team of clients, which client is at highest risk for the development of endothelial damage that may lead to disseminated intravascular coagulation (DIC)? Adolescent client who developed blisters on the back from sunbathing requiring pain medication Client on a monitor displaying frequent premature ventricular contractions (PCVS) Client with chronic obstructive pulmonary disease (COPD) who is diagnosed with pneumonia, most likely from a virus Client who had an automobile accident client and sustained three broken ribs resulting in a small pneumothorax

Former IV drug user recently diagnosed with hepatitis C virus Middle-aged client scheduled for gastroscopy to confirm Helicobactor pylori

From the current health histories, which clients) is at risk for developing secondary forms of immune thrombocytopenic purpura (ITP)? Select all that apply Former IV drug user recently diagnosed with hepatitis C virus Middle-aged client scheduled for gastroscopy to confirm Helicobactor pylori Female client in the first trimester reporting abdominal cramping back Adolescent admitted for facial burns when the "vape" (cigarette) exploded in the mouth Older adult female undergoing testing for cause of joint discomfort

The nurse volunteering in the medical tent for a road race on a hot, humid day is asked to see a runner who has collapsed on the road. The nurse notes he has sunken eyes, a temp of 100 F, and dissiness. These are the signs of a fluid volume deficit. Recognizing fluid volume deficit, which of these interventions does the nurse carry out first?

Give him an electrolyte solution by mouth

During a period of stress, the nurse asks the client to close his eyes and think of a calm, relaxing place where he can feel the wind blowing on his cheek and smell the salty air from the ocean. This is an example of utilizing which type of treatment for stress reduction?

Guided imagery

A client with Parkinson disease has challenged himself to maintain mobility for longer than the physician predicts. He strives every day to walk 5 to 10 steps farther than the day before. This phenomenon, being researched by social physcologists, is known as:

Hardiness

A client arrives in the ED reporting muscle cramps and spasms in the legs and feet; tingling in the fingers, toes, and lips; and anxiety. Serum calcium level is 4.8 mg/dL. Syspecting tetany, which medication should the nurse be prepared to administer?

IV calcium gluconate

Rh immune globulin.

If an Rh-negative mother is giving birth to an Rh-positive infant, the nurse should be prepared to administer: alpha interferon. Rh immune globulin. antihistamines like dyphenhydramine. a monoclonal antibody like infliximab.

A 21-year-old woman awaiting bone marrow transplant for myelogenous leukemia

In which client would diagnostic investigations be least likely to reveal increased thrombopoietin production? An 81-year-old woman with diagnoses of rheumatoid arthritis and failure to thrive A 66-ycar-old woman with a diagnosis of lung cancer with bone metastases A 55-year-old man with dehydration secondary to Crohn disease A 21-year-old woman awaiting bone marrow transplant for myelogenous leukemia

A 40-year-old man who has had 3 broken bones over the past 6 months and whose serum calcium and creatinine levels are elevated.

In which individuals would a clinician most suspect multiple myeloma as a diagnosis? An 81-year-old male resident of a long-term care home who has an uncommon bacterial pneumonia and who is unable to produce a fever. A 70-year-old woman whose blood work reveals large numbers of immature granulocytes. A 40-year-old man who has had 3 broken bones over the past 6 months and whose serum calcium and creatinine levels are elevated. A 68-year-old former coal miner who has white blood cell levels exponentially higher than normal ranges.

A client who has been awaiting the results of a bone marrow biopsy for several days is experiencing stress as a result of uncertainty and the possibility that abnormal cell growth may be detected. A physical examination and blood work would most likely yield which result?

Increased BP and HR; increased antidiuretic hormone (ADH)

A client with poorly controlled diabetes mellitus presents to the emergency department with suspected ketoacidosis. Which diagnostic results would be most likely to confirm this diagnosis?

Increased anion gap, base deficit

A 61 year old client with a 40 pack year history of cigarette smoking and chronic obstructive pulmonary disease (COPD) is experiencing an increase in arterial levels of CO2. This change stimulates the inspiratory center in the medulla oblongata, which in turn causes the diaphragm to contract more forcefully and increase respiratory rate. Which term best describes the role of the inspiratory center?

Integrator

A nurse is providing care for a client who has been diagnosed with metabolic alkalosis after several days of antacid use. Which treatment should the nurse prepare to give?

Intravenous administration of KCl solution

A single mother who was raised in a traumatic environment is very protective of her child. She expresses a desire to keep her child from experiencing stressful situations in hopes of promoting healthy development. What response by the nurse is most helpful?

It is normal to experience stressors periodically and this can enhance healthy development

a normal value represents the test results that fall within the bell curve.

Laboratory testing is ordered for a male client during a clinic visit for routine follow-up assessment of hypertension. When interpreting lab values, the nurse knows: -all lab values are adjusted for gender and weight. -if the result of a very sensitive test is negative, that does not mean the person is disease free. -if the lab result is above the 50% distribution, the result is considered elevated -a normal value represents the test results that fall within the bell curve.

A student is attempting to trace the feedback cycle involved in the stress response. Which neural structure is thought to be the central integrating site for the stress response?

Locus Caeruleus

A client with a diagnosis of end-stage liver failure has arterial blood gas results indicating altered pH. Which of these consequences of liver failure has most likely cause the acid-base distrubance?

Low albumin and plasma globulin levels

A client's arterial blood gases reveal normal oxygen level, pH 7.50, PCO2 level of 50 mmHg, and HCO3 level of 30. The client's respiratory rate is 12 breaths/min and all other vital signs are within normal range. What is this client's most likely diagnosis?

Metabolic alkalosis

Irreversible acetylation of platelet cyclooxygenase activity has occurred.

Misinterpreting her physician's instructions, a 69-year-old woman with a history of peripheral artery disease has been taking two 325 mg tablets of aspirin daily. How has this most likely affected her hemostatic status? Irreversible acetylation of platelet cyclooxygenase activity has occurred. The client's prostaglandin TXA2 levels are abnormally high. The binding of an antibody to platelet factor 4 produces immune complexes She is at risk of developing secondary immune thrombocvtopenic purpura (ITP).

A pregnant client having premature labor cramps and is diagnosed with preeclampsia. The admitting prescriptions include starting an IV infusion of magnesium sulfate. What intervention will the nurse provide this client?

Monitoring for changes in neuromuscular status like lethargy, confusion, or hyporeflexia

A client with a diagnosis of schizophrenia has been admitted with suspected hyponatremia after consuming copious quantities of tap water. Which finding does the nurse anticipate uncovering related to this problem?

Muscle weakness, lethargy, and headaches

The nurse is caring for a client with a longstanding diagnosis of hypocalcemia secondary to kidney disease. The nurse observes which clinical manifestations in this client?

Muscular spasms and reports of tingling in hands/feet

An 81 year old male client who has a diagnosis of orthostatic hypotension is experiencing an episode of particularly low blood pressure. The man's body has responded by increasing levels of angiotensin II in the bloodstream , a hormone which decreases glomerular filtration rate in the kidneys and contributes to an increase in blood pressure. Which phenomena best describes what has occurred?

Negative Feedback

Transitioning of hemoglobin F (HbF) to hemoglobin A (HbA)

New parents are upset their 2-day-old infant is requiring phototherapy for hyperbilirubinemia. Which factors would the pediatrician be most likely to rule out as a contributor to high bilirubin levels? The fact that the infant is being breast-fed Transitioning of hemoglobin F (HbF) to hemoglobin A (HbA) Hepatic immaturity of the infant Hypoxia

44-year-old male with a 60 pack/year smoking history who was diagnosed with a histological grade-3 lung cancer

Of the following situations, which one would be an example of a maladaptive cellular change? Group of answer choices -31-year-old marathon runner who has developed hypertrophied myocardial cells -54-year-old female who has developed ovarian atrophy following loss of estrogen stimulation during menopause -44-year-old male with a 60 pack/year smoking history who was diagnosed with a histological grade-3 lung cancer -18-year-old body builder who has developed extremely large pectoral muscles following years of weight lifting.

The nurse at a long-term care facility encourages the older adults to drink even though they may not feel thirsty at the time. Which statement supports the nurse's action?

Older adults often experience a decrease in the sensation of thirst, even when serum sodium levels are high

The nurse is caring for a client who is suffering from high levels of chronic stress. On what should the nurse focus, as a negative effect on the client's health due to chronically elevated levels of cortisol?

Osteoblast activity and protein synthesis are suppressed to refocus energy

An 80 year old woman is slated for total hip replacement the following day, and is experiencing a large amount of stress around her potential surgical outcomes. Which is most likely to be uninvolved in the physiologic response to her stress?

Parathyroid

A client with hypercalcemia has just passed a kidney stone. The nurse recognizes which of these laboratory studies should also be assessed?

Parathyroid hormone level

Erythrocyte protoporphyrin level

Parents bring their 18-month-old child to the emergency room exhibiting behavior changes and vomiting. They are concerned the child ingested something in the older home they are renovating. Laboratory findings indicate low hemoglobin and elevated creatinine. Which diagnostic test should the nurse advocate for first? -Erythrocyte protoporphyrin (EP) level -Ultrasound of the kidneys -Erythrocyte sedimentation rate -Urine test for a mercury level

A deficiency in which of these would result in an inhibition of the inflammatory response? A. Histamine B. Helper T cells C. B cells D. Vitamin K

Rationale: Histamine is a key mediator in the inflammatory system, unlike helper T cells, B cells, or vitamin K.

A 16-year-old girl has broken her arm while snowboarding and is shocked at the amount of swelling at the injury site. Which statement best explains the physiologic rationale for her swelling? A. Migration and proliferation of mast cells, neutrophils, and platelets to the injury site occupy an increased volume of tissue. B. Potent vasodilation increases the total volume of vascular space at the site of inflammation. C. Osmotic flow of plasma into the intravascular space causes increased blood volume and interstitial fluid. D. Loss of plasma proteins causes an increase in interstitial osmotic pressure.

Rationale: Swelling is the result of plasma proteins leaving the interstitial space, resulting in increased osmotic pressure of interstitial fluid and movement of fluid into tissues. Neither blood components, vasodilation, nor increased intravascular volume account for swelling.

A client has been diagnosed with a brain tumor that cannot be removed surgically. During each office visit the nurse will be assessing the client for syndrome of inappropriate antidiuretic hormone (SIADH). Which assessment would alert the clinic nurse that the client may be developing the complication?

Report of decreased urine output, no edema noted in ankles, increasing headache

In the neurotrauma unit, a teenage with a closed head injury related to an automobile accident is experiencing high intracranial pressure (ICP). He is intubated and on a ventilator. One treatment for this is to allow him to progress into which acid-base imbalance in an attempt to lower ICP?

Respiratory alkalosis

Conus Medullaris Syndrome

Several months ago, a 20 year-old male suffered a spinal cord injury brought about by a snowboard trick gone wrong. The lasting effects of his injury include flaccid bowel and bladder and the inability to obtain an erection. While sensation has been completely preserved in his legs and feet, his motor function is significantly impaired. What type of incomplete spinal cord injury has the man most likely experienced? Anterior cord syndrome • Conus medullaris syndrome Central cord syndrome Brown-Sequard syndrome

A 77 yr old female hospital client has contracted Clostridium Difficile during her stay and is experiencing severe diarrhea. Which statement best conveys a risk that this woman faces?

She is susceptible to isotonic fluid volume deficit

A 12 yr old client who experienced trauma is now having trouble dealing with many forms of conflict in the home and becomes distraught. What is the best approach for the nurse to recommend to the parents?

Talk about the past trauma often and encourage the child to think positively about the future.

An occupational health nurse working with police officers wishes to apply Selye's general adaptation syndrome theory in practice. Which concept should the nurse apply?

The alarm stage involves the release of cortisol and catecholamines

fluid imbalances

The blood work of a 44-year-old male client with a diagnosis of liver disease secondary to alcohol abuse indicates low levels of albumin. Which phenomenon would a clinician be most justified in anticipating? Acid-base imbalances Impaired thermoregulation Fluid imbalances Impaired immune function

A male client with a history of heavy alcohol use has been admitted to the hospital for malnutrition and suspected pancreatitis. The client's diagnostic workup suggests alcoholic ketoacidosis as a component of his current health problems. He is somewhat familiar with the effect that drinking has had on his nutrition and pancreas, but is wholly unfamiliar with the significance of acid-base balance. How best could his care provider explain the concept to him?

The chemical process that takes place throughout your body are thrown off very easily when your body is too acidic or not acidic enough

A counseling psychologist is working with a 30 year old female client who is experiencing the symptoms of posttraumatic stress disorder (PTSD) following a house fire several months prior. Which of the client's diagnostic results could most likely be interpreted as a manifestation of PTSD?

The client has decreased levels of cortisol

A 56 yr old female hospital client with a history of alcohol abuse is receiving IV phosphate replacement. Which health problem will this IV therapy most likely resolve?

The client is acidotic and has impaired platelet function

The guidelines will combine individual expertise with external systematic evidence

The clinical educator of a hospital medical unit has the mandate of establishing evidence-based practice guidelines for the nursing care on the unit. Which statement most accurately captures a guiding principle of the nurse's task? -The need for continuity and standardization of guidelines will mean that they will be fixed rather than changeable. -Guidelines are synonymous with systematic research reviews. -Evidence-based practice guidelines will be rooted in research rather than nurses' subjective practice preferences and experiences -The guidelines will combine individual expertise with external systematic evidence.

( Sit client in an upright position. Remove compression stockings. Assess recent urine output.

The clinician is caring for a client who suffered a spinal injury at T4 several years ago. The client develops a flushed neck, reports feeling unwell, and has an elevated blood pressure. What are the clinician's priority actions? Select all that apply. ( Sit client in an upright position. Remove compression stockings. Insert large bore intravenous catheter. Administer oxygen via face mask. • Assess recent urine output.

Your medication needs to be taken at equal intervals to reduce symptom fluctuation.

The clinician is caring for client who has been living with Parkinson disease for the past 10 years and is being treated with levodopa-carbidopa. The client often leaves the unit with family for extended periods. What should be prioritized when teaching the client? Due to your risk for falls it is recommended you do not leave the unit with family Frequent assessments are needed to prevent complications related to your condition. It is essential that you eat a healthy diet with adequate amounts of protein if dining out. Your medication needs to be taken at equal intervals to reduce symptom fluctuation.

When explaining to a client admitted for stress-induced supraventricular tachycardia, the nurse will incorporate which statement about what happens in the body as a result of excess stress?

The corticotrophin-releasing factor stimulates the release of norepinephrine, which is responsible for "fight-or-flight" reaction to the stress

"In this case, the radiation is being used to help alleviate the bone pain your family member is experiencing.

The daughter of a client with metastatic cancer is confused as to why the client is receiving radiation therapy and asks the nurse, *Why are we still treating the cancer? The plan was only for comfort care. What is the nurse's best response? -This type of radiation has no serious adverse effects so we often apply it in palliative cases as there is no harm -"Radiotherapy is slowing tumor growth so you can have more time with your family member, but it is not curative." -*You will need to discuss the plan of care with your family member as I am not at liberty to discuss it with you!' back -"In this case, the radiation is being used to help alleviate the bone pain your family member is experiencing.

An electrician who has been working 14 to 16 hour days for several weeks to ensure the financial survival of his business presents to his family physician with a report of persistent headaches and insomnia. His family physician attributes the physical symptoms to the ongoing stress likely caused by which component of the stress response?

The fact that he has previously had difficulty coping with stress

"This weight loss is related to the cancer itself and occurs despite an intake of adequate calories."

The family member of a client with terminal metastatic cancer who is experiencing cachexia-related weight loss asks the nurse why the client is losing weight despite taking in a large amount of calories per day. What is the nurse's best response? -When clients are at this stage of metastatic cancer the food being caten is not being digested.! -*We can consult with the doctor and dictitian about tube feeding to prevent more weight loss. -"This weight loss is related to the cancer itself and occurs despite an intake of adequate calories." -We likely need to increase the calories mores you are welcome to bring food from home."

Products of the tumor itself as well as a hypermetabolic state cause cachexia.

The family of a 68-year-old man who is in the end stages of small-cell lung cancer are distraught at his visible body wasting that has worsened in recent weeks. Which phenomena best accounts for the client's anorexia and cachexia? -High-fat losses coupled with preservation of muscle mass exaggerate the apparance of wasting. -Products of the tumor itself as well as a hypermetabolic state cause cachexia. -Inadequate cellular metabolism of glucose results from tumor factors. -Inadequate food intake due to symptoms and treatment results in loss of both muscle and fat.

"Due to stress, the red blood cells of older adults are not replaced as promptly as younger people."

The family of an older adult client is wondering why his "blood counts" are not rising after his last GI bleed. They state, "He has always bounced back after one of these episodes, but this time it isn't happening. Do you know why?" The nurse will respond based on which pathophysiologic principle? "Everything slows down when you get older. You just have to wait and see what happens." "Due to stress, the red blood cells of older adults are not replaced as promptly as younger people." "The doctor may start looking for another cause of his anemia, maybe cancer of the bone." "Don't worry about it. We can always give him more blood."

Increasing the functional ability of the underactive dopaminergic system

The geriatrician providing care for a 74-year-old man with diagnosis of Parkinson disease has recently changed the client's medication regimen. What is the most likely focus of the pharmacologic treatment of the man's health problem? Preventing axonal degradation of motor neurons 6 Increasing the functional ability of the underactive dopaminergic system Preventing demyelization of the efferent cerebellar pathways Maximizing acetylcholine relcase from synaptic vesicles at neuromuscular junctions

low blood counts, excessive diarrhea

The health care provider is discussing the treatment protocol using ionizing radiation to treat the client's cancer. The client asks, "What side effects can occur with this treatment?" Which response(s) is accurate? Select all that apply. Photosensitivity Low blood counts New-onset seizures Kidney stone formation Excessive diarrhea

Presence of thrombocytopenia If the client has anemia The presence of leukocytosis

The health care provider is ordering a complete blood count (CBC). What information should the nurse expect to be able to gather from the results? Select all that apply. Presence of thrombocytopenia If the client has anemia If the client is deficient in clotting factors The presence of leukocytosis If there are common vitamin deficiencies

A 77 yr old client diagnosed with chronic obstructive pulmonary disease (COPD) is experiencing impaired gas exchange and CO2 retention, despite a rapid respiratory rate. Which pathophysiologic principle would the health care team expect if the client's compensatory mechanisms are working?

The kidneys will adapt with an increase in plasma HCO3 and the pH will increase

High sensitivity, low specificity

The laboratory technologists are a discussing a new blood test that helps establish a differential diagnosis between shortness of breath with a cardiac etiology and shortness of breath with a respiratory/pulmonary etiology. A positive result is purported to indicate a cardiac etiology. The marketers of the test report that 99.8% of clients who have confirmed cardiac etiologies test positive in the test. However, 1.3% of clients who do not have cardiac etiologies for their shortness of breath also test positive. Which statement best characterizes this blood test? -High specificity; low reliability -High sensitivity; low reliability -High sensitivity, low specificity -Low validity; high reliability

A female client with a new diagnosis of systemic lupus erythematosus (SLE) has been told that this is an autoimmune disease whereby the immune system is attacking the body's cells and tissue. She knows that she has inflammation and tissue damage. She asks her to explain What cells in the body are triggering this inflammation? The nurse responds:

The lymphocytes that migrate to the brain where they secrete cytokines, which trigger inflammation

Large, protruding abdomen Weight loss Crying when joints/position changed

The mother of an 18-month-old child is concerned that her child is lethargic and not eating foods that he normally enjoys. She takes him to the pediatrician for a check-up. Which clinical manifestations lead the health care provider to suspect the child may have a neuroblastoma? Select all that apply. Large, protruding abdomen Excessive burping Weight loss Crying when joints/position changed Large amount of pale urine

-The development of evidence-based practice guidelines require a research review from different studies to develop the most accurate diagnostic method to implement. -A meta-analysis could be utilized to combine evidence from different studies to produce a more accurate diagnostic method. -When developing a CVA set of step-by-step directions, the nursing unit should ask for assistance from experts in the neuroscience field. The potential users of the guidelines should pilot test it for further feedback.

The neuroscience nursing unit has developed a set of step-by-step directions of what should occur if a nursing assessment reveals the client may be exhibiting clinical manifestations of a cerebrovascular accident (CVA). Which statement about clinical practice guidelines are accurate? Select all that apply. -Step-by-step guidelines are usually developed and based primarily on "how it has always been done before.' -Once developed, practice guidelines only need to be reviewed if a national committee sends out an update on new research. -The development of evidence-based practice guidelines require a research review from different studies to develop the most accurate diagnostic method to implement. -A meta-analysis could be utilized to combine evidence from different studies to produce a more accurate diagnostic method. -When developing a CVA set of step-by-step directions, the nursing unit should ask for assistance from experts in the neuroscience field. The potential users of the guidelines should pilot test for further testing

Serum sodium of 115 mEq/L (115 mmol/L)

The nurse caring for a lung cancer client with metastasis to the brain suspects the client has developed a paraneoplastic syndrome known as Syndrome of Inappropriate Antidiuretic Hormone (SIADH) secretion. Which laboratory result in this client who has gained 3 lb in a day would alert the nurse to the possibility of SIADH? -Serum sodium of 115 mEq/L (115 mmol/L) -BUN of 8 mg/dL (2.86 mmol/L) -Serum potassium of 5.0 mEa/L (5.0 mmol/L) -Hematocrit of 40%

Dyspnea on exertion oncave fingernails (koilonychia) Oral lesions

The nurse is assessing a client. What findings should the nurse apply as evidence of iron deficiency anemia? Select all that apply. Jaundice Dyspnea on exertion Paresthesia of the feet and fingers Concave fingernails (koilonychia) Oral lesions

This iron deficiency anemia was most likely the result of chronic blood loss over a period of time.

The nurse is caring for a 30-year-old woman who is diagnosed with iron deficiency anemia. The client is confused how this condition developed because she ate many foods high in iron. How should the nurse respond? Diet deficiencies are the most common reason, so it is likely the intake was not sufficient. Some people have a condition that impairs the ability to use the iron they take in effectively. Iron deficiency anemia is actually more likely linked to deficits of vitamin B12 and folate in the diet. This iron deficiency anemia was most likely the result of chronic blood loss over a period of time.

Avoiding invasive procedures Performing respiratory assessment Assessing for fatigue

The nurse is caring for a 45-year-old client undergoing radiotherapy of the mediastinal nodes due to lymphoma. Which interventions should the nurse prioritize? Select all that apply. -Avoiding invasive procedures -Performing respiratory assessment -Assessing for fatigue -Performing neurologic assessment -Assessing for urinary retention

Interview the client regarding side effects of corticosteroid therapy.

The nurse is caring for a client who has been on an oral corticosteroid for several years for an autoimmune condition. The nurse notes several bruises of different stages of resolution on the client. What is the nurse's best response to this finding? Interview the client regarding side effects of corticosteroid therapy. Obtain a STAT order for repeat platelet count and international normalized ratio (INR). Complete a thorough pain assessment related to the bruising. Request an order for vitamin K to prevent life-threatening hemorrhage.

Bence Jones proteins in urine 20% plasma cells in bone marrow biopsy X-rays demonstrate bone loss

The nurse is caring for a client who is suspected to have multiple myeloma. What findings will the nurse use as evidence the client has multiple myeloma? Select all that apply. Bence Jones proteins in urine 20% plasma cells in bone marrow biopsy Erythrocytosis Hyperkalemia X-rays demonstrate bone loss

Whether the medication is lipid or water-soluble

The nurse is caring for a client who was just told she is 4 months pregnant and was unaware of the pregnancy. The nurse is reviewing any medications that the client may have taken in the past 5 months. Which characteristic of the medication(s) will the nurse assign the most significance when considering if the medication may be teratogenic? •How long the client took the drug prior to becoming pregnant •If the drug was a prescribed or taken without prescription •The route of the medication (oral, transdermal, or parenteral) • Whether the medication is lipid or water-soluble

drop in blood pressure from 130/90 to 95/50 mm Hg.

The nurse is caring for a client with Marfan syndrome. Which assessment finding should the nurse prioritize as requiring the most emergent intervention? A sudden: •drop in heart rate from 80 to 60 beats per minute. •onset of confusion and altered level of consciousness. • drop in blood pressure from 130/90 to 95/50 mm Hg. •complete loss of vision in a single eye.

diminished pulse strength atrophy to affected toes a clear demarcation between healthy and affected tissue -darkened appearance of affected tissue

The nurse is caring for a client with arterial insufficiency of the left leg with gangrenous wounds on the second and third toes. What characteristics of the wounds should the nurse evaluate as expected? Select all that apply. -Diminished pulse strength -Atrophy to the affected toes -Moist wound with evidence of liquefaction -A clear demarcation between the healthy and affected tissue -Edema to the left lower leg -Darkened appearance of affected tissue

Elevated unconjugated bilirubin

The nurse is caring for a client with jaundice. What evidence will the nurse use to determine if the cause of the jaundice is due to a hemolytic condition? Decreased conjugated bilirubin Elevated unconjugated bilirubin Liver function tests Decreased hemoglobin

Headache New onset dyspnea Lethargy

The nurse is caring for a client with leukemia and a blast count of 100,000 cells/μL (100 x 109/L). What client reports should prompt the nurse to take immediate action? Select all that apply. Headache New onset dyspnea Lethargy Increased urine output Bleeding from gums

Assist for ambulation Encourage increased fluid intake Monitor urine output

The nurse is caring for a client with multiple myeloma. What aspects of care should the nurse prioritize? Select all that apply. Restrict to a soft diet Position in high Fowler's Assist for ambulation Encourage increased fluid intake Monitor urine output

Level of consciousness Oxygen saturation Urine output

The nurse is caring for a client with sepsis who has developed disseminated intravascular coagulation (DIC). Which assessments should the nurse prioritize? Select all that apply. Level of consciousness Weight Oxygen saturation Urine output Platelet count

Daily weights Maintenance of fluid restriction Frequent repositioning

The nurse is caring for a client with small cell lung carcinoma who has developed syndrome of inappropriate antidiuretic hormone (SIADH). What should the nurse incorporate into the plan of care? Select all that apply. -Daily weights -Maintenance of fluid restriction -Administration of hypotonic intravenous solutions -Restriction of sodium in the diet -Frequent repositioning

Accelerated intravascular hemolvsis Hypoperfusion of the liver

The nurse is caring for a client with thrombotic thrombocytopenic purpura (TTP) and notices a yellow discoloration of the client's sclera. Which are likely possible causes of this finding? Select all that apply. Side effect of hepatotoxic medication Viral hepatitis Elevated levels of nitrogenous wastes (uremia) Hypoperfusion of the liver Accelerated intravascular hemolysis

PaCO2 of 50 mm Hg (6.65 kPa) I Pa02 of 70 mm Hg (9.31 KPa) pH of 7.25 • PaCO2 of 28 mm Hg (3.72 kPa)

The nurse is caring for a client with traumatic brain injury and increased intracranial pressure. What findings should the nurse report due to the risk for increasing intracranial pressure? Select all that apply. PaCO2 of 50 mm Hg (6.65 kPa) I Pa02 of 70 mm Hg (9.31 KPa) pH of 7.25 • PaCO2 of 28 mm Hg (3.72 kPa) potassium of 3.3 mEg/L (3.3 mmolL)

Elevated serum osmolality Elevated white blood cell count Low pH on an arterial blood gas

The nurse is caring for a teenager with sickle cell anemia. When comparing new laboratory results with the client's baseline, which results should the nurse prioritize as requiring intervention? Select all that apply. Decreased hemoglobin Elevated serum osmolality Elevated serum bilirubin Elevated white blood cell count Low pH on an arterial blood gas

If you have another son there is a 50% chance that he will also have hemophilia.

The nurse is caring for an 8-month-old male client who has just been diagnosed with hemophilia A. The parents ask about the likelihood that any future children would be born with the condition. What is the nurse's best response? •If you have another son there is a 50% chance that he will also have hemophilia. •I cannot offer an opinion as the pattern of inheritance is unknown and you need to seek genetic counseling •Since you have already had a son with hemophilia, the chance that you will have another child with the condition is unlikely. •There is a 50% chance that your next child will have hemophilia but the severity varies and it could be a mild form.

Lethargy Reduction in feeding Rigidity

The nurse is caring for neonates undergoing treatment for hyperbilirubinemia. Which assessment findings should the nurse prioritize? Select all that apply. Lethargy Jaundice Increase in urine output Reduction in feeding Rigidity

invert the tube several times to prevent the blood from coagulating.

The nurse is collecting a blood sample for an ordered CBC. The required tube contains EDTA so the nurse will: Avoid inverting the tube to prevent hemolysis, which could affect the accuracy of the results place the tube immediately on ice to preserve the action of the EDTA on the sample. invert the tube several times to prevent the blood from coagulating. Invert the tube several times to promote clotting so serum can be fully separated

10

The nurse is counseling a client who is going for major surgery and must stop taking aspirin to reduce the risk for bleeding. The nurse knows aspirin decreases platelet activity for the duration of the platelet's lifespan. How many days prior to the surgery should the nurse tell the client to stop taking aspirin? 14 2 10 5

less cognitive dysfunction

The nurse is educating a client who is undergoing gamma knife radiosurgery for a brain metastasis. What advantage to this procedure should the nurse share when comparing it to other conventional treatments? -lessen the number of migraine headadhes per day -less cognitive dysfunction -less explosive vomiting -less chance of developing focal seizures

We want to ensure your shoulder joint remains flexible while you recover. These exercises help to prevent painful shoulder complications after a stroke.

The nurse is performing passive range-of-motion exercises with a client who suffered an ischemic stroke 2 weeks ago and has hemiparesis. The client says, "What is the point of doing these exercises if I will never be able to use that arm again?" What should the nurse include when responding to the client? Select all that apply. Doing these exercises helps to prevent blood clots from forming in the arteries of your arm. You may still regain use of this arm, and these exercises may help with this recovery. It is important for you to keep a positive outlook as it will help with your recovery. We want to ensure your shoulder joint remains flexible while you recoven These exercises help to prevent painful shoulder complications after a stroke.

Adequate nutrition

The nurse is planning care for a 6-hour-old neonate who has been born with cleft palate. What aspect of care should the nurse prioritize? • Adequate nutrition •Managed parental anxiety •Infection control •Positive body image

swallowing precautions falls prevention deep vein prophylaxis

The nurse is planning care for a client in the acute recovery phase after an ischemic stroke. What interventions will the nurse prioritize when planning care? Select all that apply. Falls prevention Reorientation exercises Deep vein prophylaxis Stroke prevention education Swallowing precautions

Falls prevention protocol Monitor bowel movements closely. • Daily weights

The nurse is planning care for a client who is receiving antimetabolite chemotherapy. What should the nurse include in the plan of care? Select all that apply. Indwelling urinary catheter for accurate output record Encourage intake of a soft diet with fresh fruits and vegetables Falls prevention protocol Monitor bowel movements closely. Daily weights

A 25-year-old client with an absolute neutrophil count of 300/mm3 (0.30 x 109/L)

The nurse is planning care for a group of clients. Which client should the nurse assess first? A 25-year-old client with an absolute neutrophil count of 300/mm3 (0.30 x 109/L) A 65-year-old client with a white blood cell count of 15,200/μL (15.2 x 109/L) A 45-year-old client with a platelet count of 100,000/mm3 (100 x 109/L) An 82-year-old client with a hemoglobin of 9.0 g/dL (90 g/L)

A client with prolonged immobility A client who has breast cancer with bone metastasis A client with hypophosphatemia

The nurse is planning care for a group of clients. Which clients should the nurse prioritize as "at risk" for hypercalcemia and advocate for monitoring calcium levels? Select all that apply. -A client with prolonged immobility -A client who has breast cancer with bone metastasis -A client with hypophosphatemia -A client with hypoparathyroidism -A client with heart failure who is taking a loop diuretic

"This drug helps reduce uric acid levels and protect your kidneys."

The nurse is preparing to administer rasburicase to a client undergoing chemotherapy. How should the nurse explain the purpose of this medication to the client? "Chemotherapy causes high phosphate and this medication reduces this." "Rasburicase is a unique antiemetic that will reduce your nausea." "This drug helps reduce uric acid levels and protect your kidneys." "This medication protects your bladder from the toxic effects of chemotherapy."

apoptosis

The nurse is providing care for a client with a diagnosis of amyotrophic lateral sclerosis (ALS). The nurse recognizes that which mechanism is suspected to play a role in the cellular death associated with ALS? -Hypoxic cell injury -Caseous necrosis -Liquefaction necrosis -Apoptosis

Compare clients' results to measurements taken using arterial blood gas analyses.

The nurse is questioning the validity of oxygen saturation readings from the new oximeters on the unit. Which action should the nurse take to best determine if the results from the oximeter are valid? •Compare the findings with all clients' results on the unit to determine if there is a trend in the results •Repeat the test on the same client to determine if the oximeter offers the same results •Review the literature about this brand of oximeter provided by the company. •Compare clients' results to measurements taken using arterial blood gas analyses.

"Transfusions are needed to prevent skeletal abnormalities from developing." "A stem cell transplant may be used as part of the treatment plan."

The nurse is reviewing the treatment plan with parents of a 6-month-old diagnosed with severe β-thalassemia. Which statements by the parents demonstrate they understand the treatment plan? Select all that apply. "Transfusions will need to begin once our child reaches 2 years of age." "Transfusions are needed to prevent skeletal abnormalities from developing." "We will need to feed our child a diet high in iron to promote red blood cell production." "A stem cell transplant may be used as part of the treatment plan." Oxygen therapy should be used to prevent organ damage from chronic hypoxia.

there are some modest risks, but they are only associated with some long living fish like tuna

The nurse is teaching a group of new mothers about postpartum nutrition, when one of the clients states that she was told to avoid eating fish too often due to the risk of mercury poisoning. Which response by the nurse most accurately addresses the clients concerns? =-"Provided you avoid salmon, you likely won't be putting yourself or your child at risk." -"There are some modest risks, but they are only associated with some long-living fish like tuna." -"You're right. It's best to avoid eating fish, especially while you are breast-feeding." -"The risk of mercury toxicity from eating fish has been shown to be insignificant."

hypertension, obesity, and decreased activity tolerance

The nurse is teaching new nursing assistants on the unit about the phenomenon of muscle hypertrophy. Which client on the unit is most likely to experience muscle hypertrophy? A client with: -hypertension, obesity, and decreased activity tolerance. -possible rejection symptoms following a liver transplant. -peripheral edema secondary to heart failure (HF) -urinary incontinence following a cerebrovascular accident (CVA).

asthma, exposure to parasites, allergic reactions

The nurse notes eosinophilia on the client's laboratory results. The nurse should interview the client about a history of which conditions? Select all that apply. Exposure to parasites Asthma Allergic reactions Viral infection Bacterial infection

interferon beta

The nurse should anticipate that she will need to teach the client newly diagnosed with multiple sclerosis how to give injections if which medication to assist with reducing exacerbations is prescribed? Mitoxantrone Corticosteroids Plasmapheresis Interferon beta

Reports of having a hard time concentrating in school Speech impediments Severe scoliosis

The nurse working in a pediatric office is scheduled to assess a female adolescent diagnosed with neurofibromatosis (NF) Type 1. During this assessment, the nurse should be assessing the teenager for which clinical manifestations of NF-1? Select all that apply. -irregular menstrual periods -Hearing loss -Reports of having a hard time concentrating in school -Speech impediments -Severe scoliosis

Which physiologic process would be considered a positive feedback mechanism?

The platelet-aggregation mechanism activating clot formation

Cardiovascular disease

The public health nurse is designing a course about risk factors for various chronic illnesses. For risk factors about which chronic illness will the nurse consult the Framingham study? •Cardiovascular disease • Type 2 diabetes mellitus •Chronic obstructive pulmonary disease (COPD' •Breast cancer

notify parents to pick up the child and possibly administer Factor VIII.

The school nurse has several children with hemophilia A. After recess, one hemophilia student comes to the school nurse complaining of pain in the knee from falling on the playground. The nurse notes there is swelling in the knee and pain on palpation. The nurse should: notify parents to pick up the child and possibly administer Factor VIII. administer some NSAIDs to relieve the pain. apply some warm compresses to the knee. wrap the knee in an ace bandage for compression

"Have you ever given your child any honey or honey-containing products?'

The unique clinical presentation of a 3 month-old infant in the emergency department leads the care team to suspect botulism. Which assessment question posed to the parents is likely to be most useful in the differential diagnosis? "Have you ever given your child any honey or honey-containing products?' Has your baby ever been directly exposed to any chemical cleaning products? *Is there any family history of neuromuscular diseases?" Is there any mold in your home that you know of?'

"A couple with a child with a multifactorial disorder have a higher risk of having another with the same disorder."

Two health care workers are comparing the etiology and incidence of multifactorial inheritance disorders and single-gene disorders. Which statement best captures the relationship between the two types of genetic disorders? -Multifactorial disorders manifest themselves at birth -Multifactorial disorders and single-gene disorders can both be predicted quite accurately." -"A couple with a child with a multifactorial disorder have a higher risk of having another with the same disorder." -"Multifactorial disorders are more likely to involve multiple organs.

"Both of them involve low platelet counts, but in TTP there can be more, not less, hemostasis

Two nursing students are attempting to differentiate between the presentations of immune thrombocytopenic purpura (ITP) and thrombotic thrombocytopenic purpura (TTP). Which of the students' statements best captures an aspect of the two health problems? "Both diseases can result from inadequate production of thrombopoietin by megakaryocytes. TTP can be treated with plasmapheresis, but IT is best addressed with transfusion of fresh frozen plasma." "Both of them involve low platelet counts, but in TTP there can be more, not less, hemostasis 'ITP can be either inherited or acquired, and if it's acquired, it involves an enzyme deficiency.

Complaints of weakness and fatigue Small spots of skin hemorrhages over entire body Excess bleeding from gums and nose

Two years after chemotherapy and radiation therapy for lung cancer, a 72-year-old client reports being extremely tired all the time. The physician suspects the client may have developed aplastic anemia. The nurse assessing the client will likely find which clinical manifestation of aplastic anemia? Select all that apply. Hemolysis from renal dialysis treatments Spoon-shaped deformity of the fingernails Complaints of weakness and fatigue Small spots of skin hemorrhages over entire body Excess bleeding from gums and nose

epidemic mechanisms

Unbeknownst to her or her care team, a 51-year-old woman's breast cancer has an etiology rooted in the fact that tumor suppressing genes are present but have been silenced. Consequently, she has not synthesized normal cancer-suppressing proteins and neoplasia has resulted. What process has accounted for the woman's cancer? • Epigenetic mechanisms •The "two-hit" hypothesis of carcinogenesis •A DNA repair defect. •Chromosomal translocation

Discuss an intrauterine transfusion to the affected fetus.

When a Rh-negative mother becomes pregnant by a partner who is Rh-positive, which advanced treatment modality will decrease the risk of the fetus developing hemolytic disease? Discuss an intrauterine transfusion to the affected fetus. Discuss options to prevent any further pregnancy. Give mother gamma globuline-containing Rh antibody in the first trimester of pregnancy. Perform aminocentesis to draw unconjugated bilirubin and immediate delivery if levels are high.

plasminogen

When discussing the sequence of clot dissolution, the science instructor will talk about which item that begins the process? Dabigatran Platelets Plasminogen a2-plasmin inhibitor

infants through breast milk blood by sharing needles sexual intercourse

When educating a client who has tested positive for human T-cell leukemia virus-1 (HTLV-1), what mode(s) of transmission should the nurse discuss to prevent the spread? Select all tht apply. -sneezing (releasing droplets) -infants through breast milk -kissing a person on the forehead - blood by sharing needles -sexual intercourse

shaped like a sphere with multilobar nuclei

When looking at a granulocyte under a microscope, the anatomy student would describe it as: having no nuclei. shaped like a sphere with multilobar nuclei. lacking granules. having a kidney-shaped nucleus.

4 days

When talking about the lifespan of various blood components, the students should know that once a neutrophil moves into tissue, it lives approximately how long? 12 hours. 4 days. 24 hours. 2 days.

basophils

When talking about the various types of granulocytes, which granule contains heparin, an anticoagulant? Neutrophils Lymphocytes. Basophils. Eosinophils.

High blast cell counts and fever

Which assessment and laboratory findings would be most closely associated with acute leukemia? High blast cell counts and fever Decreased oxygen partial pressure and weight loss Increased serum potassium and sodium levels Increased blood urea nitrogen and bone pain

Elevated temperature Night sweats Weight loss

Which assessment finding(s) correlatea with the health care providers preliminary diagnosis of a more aggressive form of non-Hodgkin lymphoma (NHL)? Select all that apply. Elevated temperature Enlarged pancreas Night sweats Reed-Sternberg cells in lymph node tissue Weight loss Peripheral edema

Sausage fried in reused grease and biscuits and gravy.

Which breakfast diet contains chemicals that likely can contribute to a higher rate of cancer risk? -Sausage fried in reused grease and biscuits and gravy. -English muffin with jelly and scrampled eggs. -Tofu stir-fried with peppers and onions and topped with cheddar cheese. -Pancakes made with buttermilk and warm maple syrup

37-year-old client with leukemia being treated with chemotherapy with ANC of 400 (0.40 x 109/L).

Which client has an absolute neutrophil count (ANC) that is critically low and the standard of care would recommend placement on neutropenic precautions? 65-year-old client with prostate cancer receiving radiation therapy with neutrophil count of 2,000/μL (2.0 x 109/L). 37-year-old client with leukemia being treated with chemotherapy with ANC of 400 (0.40 x 109/L). 75-year-old client with renal failure receiving epogen for anemia with hemoglobin level of 9.7 g/dL (97 g/L). Client on long-term steroids for rheumatoid arthritis with WBC of 7000/μL (7 x 109/L).

44 year old noncompliant female who forgets to take her hypertensive medications.

Which client is at high risk for developing dilated cardiomyopathy? -78-year-old Alzheimer client who received a third-degree burn following an oven fire -4-year-old child born with cerebral palsy and confined to a wheelchair -44-year-old noncompliant female who forgets to take her hypertensive medications -17-year-old with a diving injury resulting in paraplegia

56-year-old reporting tingling sensations and has both an elevated pulse and BP

Which client may be experiencing a sensory focal seizure that has sent an abnormal cortical discharge to the autonomic nervous system (ANS)? 56-year-old reporting tingling sensations and has both an elevated pulse and BP 22-year-old reporting a stiff neck and achiness, along with some nausea and vomiting 44-year-old client reporting constant movement and pain in the legs that gets worse when attempting to sleep 85-year-old client experiencing drooping of the right side of face and numbness in right arm and leg

A 38-year-old female with Down syndrome and congenital scoliosIs

Which client of a primary care physician would not require extra screening for cancer? -A 50-vear-old male who is obese and has a low-fiber. high-fat diet. -A 38-year-old female with Down syndrome and congenital scoliosIs -A 48-year-old man who takes immunosuppressant drugs following a kidney transplan -A 51-year-old woman whose grandmother died of breast cancer

A 71-year-old smoker admitted to hospital with exacerbation of his chronic obstructive pulmonary disease (COPD)

Which client would be most likely to be experiencing an increase in renal erythropoietin production? A 68-year-old man with a long-standing diagnosis of polycythemia vera A 71-year-old smoker admitted to hospital with exacerbation of his chronic obstructive pulmonary disease (COPD) A 21-year-old man with acute blood loss secondary to a motor vehicle accident 3 hours prior A 70-year-old woman admitted with dehydration secondary to an overdose of her potassium-wasting diuretic

has had a gradual onset of weakness, headache, and visual disturbances over the last 2 days.

Which client's signs and symptoms would allow a clinician to be most justified in ruling out stroke as a cause? An adult: has vomited and complained of a severe headache states that her left arm and leg are numb and gait is consequently unsteady experienced a sudden loss of balance and slurred speech. 7 has had a gradual onset of weakness, headache, and visual disturbances over the last 2 days.

55-year-old client diagnosed with Cushing disease displaying bruises, weight gain with a buffalo hump, and "moon" face Child adopted from India and displaying malaise, lethargy, and petechia all over the body associated with suspected scurvy.

Which clients would be at risk for developing nonthrombocytopenic purpura? Select all that apply. 15-year-old with insulin-dependent diabetes who has hypoglycemia and is displaying irritability with headaches and tachycardia 55-year-old client diagnosed with Cushing disease displaying bruises, weight gain with a buffalo hump, and "moon" face Pregnant mother experiencing headaches and proteinuria 73-year-old client admitted with concussion that resulted from a fall Child adopted from India and displaying malaise, lethargy, and petechia all over the body associated with suspected scurvy.

Mean arterial pressure (MAP) that equals intracranial pressure (CP)

Which diagnostic finding is likely to result in the most serious brain insult? Mean arterial pressure (MAP) that equals intracranial pressure (CP) High intracellular concentration of glutamate 17 Moderate decrease in brain tissue volume secondary to a brain tumor removal Increased IC accompanied by hyperventilation

telomerase

Which enzyme listed below is responsible for the ability of cancer cells to resist aging and contributes to cellular immortality that is so characteristic of this disease process? -Telomerase -Hydrolase -Oxidoreductase -Isomerase

Growth factors and cytokines.

Which glycoprotein is responsible for treating such diseases as bone marrow failure following chemotherapy and hematopoietic neoplasms such as leukemia? T lymphocytes and natural killer cells. Growth factors and cytokines. Neutrophils and eosinophils. Natural killer cells and granulocytes.

who had his forearm partially crushed by gears during an industrial accident.

Which individual is likely to have the best prognosis for recovery from his or her insult to the peripheral nervous system? An adult: who suffered a bone-depth laceration to the shoulder during a knife attack client who developed rhabdomyolysis and ischemic injury after a tourniquet application client who had nerves transected during surgery to remove a tumor from the mandible. who had his forearm partially crushed by gears during an industrial accident.

A man who has entered cardiogenic shock following a severe myocardial infarction

Which individual would be most likely to experience global ischemia to his or her brain? A man who has entered cardiogenic shock following a severe myocardial infarction A woman who is being brought to the hospital by ambulance following suspected carbon monoxide poisoning related to a faulty portable heater A woman who has been admitted to the emergency department with a suspected intracranial bleed A male client who has just had an ischemic stroke confirmed by CT of his head

A 78-year-old male who has been diagnosed with chronic lymphocytic leukemia (CLL).

Which individuals would most likely possess normal plasma cell synthesis and fully differentiated myeloid and lymphoid cells? A 78-year-old male who has been diagnosed with chronic lymphocytic leukemia (CLL). A 7-year-old boy with a diagnosis of acute lymphocytic leukemia (ALL). A 58-year-old female with HIV and multiple myeloma A 70-year-old male who has acute myelogenous leukemia (AML)

A woman who has herpes simplex and who has recently recovered from endocarditis.

Which pregnant woman has most likely encountered the greatest increase in the risk that her child will have a fetal anomaly? -A woman who has herpes simplex and who has recently recovered from endocarditis. -A woman with chronic obstructive pulmonary disease (COPD) and tuberculosis. -A woman with diagnoses of insulin-dependent diabetes mellitus and peripheral neuropathy. -A woman with diagnoses of syphilis and cirrhosis of the liver.

-Massive pulmonary emboli following diagnosis of new onset atrial fibrillation -Development of pulmonary fibrosis following treatment with bleomycin, an antibiotic chemotherapy agent used in treatment of lymphoma.

Which situation would be classified as a complication of a disease or outcome from the treatment regimen? Select all that apply. -Burning, intense incision pain following surgery to remove a portion of colon due to intestinal aganglionosis. -Massive pulmonary emboli following diagnosis of new onset atrial fibrillation Gradual deterioration in ability to walk unassisted for a client diagnosed with Parkinson disease. -Development of pulmonary fibrosis following treatment with bleomycin, an antibiotic chemotherapy agent used in treatment of lymphoma. -Loss of short-term memory in a client diagnosed with Alzheimer disease

They contribute to the maintenance of blood pH

Which statement best conveys a characteristic of red blood cells? They help maintain the body's fluid balance. They lack organelles and soluble enzymes. They contribute to the maintenance of blood pH They are self-replicating.

They confirmed my diagnosis with a lymph node biopsy and I'll get radiation treatment soon because it's fairly early stage."

Which statement by a client who has a new diagnosis of non-Hodgkin lymphoma (NHL) demonstrates a sound understanding of the diagnosis and treatment of the health problem? "Since the tests show NHL, I'm going to pursue my options for palliative care because I'm committed to dying with dignity." "They took a sample of my lymph nodes and I'll be having surgery soon that will hopefully cure my lymphoma." "They confirmed my diagnosis with a lymph node biopsy and I'll get radiation treatment soon because it's fairly early stage." "My blood work came back positive for NHL, and I'm meeting with my oncologist to discuss chemotherapy soon."

inured cells tend to accumulate calcium

Which statement most accurately conveys an aspect of cell injury due to impaired calcium homeostasis? -Low calcium levels cause an activation of damaging enzymes. -Ischemia and certain toxins cause a decrease in cytosolic calcium. -Normal intracellular calcium ion levels are higher than extracellular levels. -Injured cells tend to accumulate calcium.

B and T lymphocyte development begins in the bone marrow and ends in the peripheral lymphoid structures.

Which statement most accurately conveys an aspect of lymphatic system activity? B cells and macrophages are released from the bone marrow in their completed state. Stem cells in the lymph nodes initiate and regulate the process of white cell synthesis. B and T lymphocyte development begins in the bone marrow and ends in the peripheral lymphoid structures. Leukocytes bypass vascular circulation and are distributed instead by the lymphatic system.

Progenitor cells differentiate into precursor cells.

Which statements best captures an aspect of the process of hematopoiesis? Colony-stimulating factors (CSFs) produce cytokines that activate progenitor cells. Various subtypes of pluripotent stem cells eventually differentiate into the cellular components of blood. Progenitor cells differentiate into precursor cells. Self-replicating precursor cells differentiate into specific CSFs.

"I know that cells like neurons have little capacity for hyperplastic growth."

Which student statement demonstrates a sound understanding of the cellular processes of hypertrophy and hyperplasia? -"When male clients experience 'an enlarged prostate,' they are describing a form of hypertrophy." -"A remaining kidney can sometimes undergo hyperplasia in response to one that has been removed." -"Clients with cardiomyopathy undergo myocardial hypertrophy with proportional increases in cell length and width." -"I know that cells like neurons have little capacity for hyperplastic growth."

"'Make sure that you avoid taking aspirin."

Which teaching point would be most appropriate with a client who has a recent diagnosis of Von Willebrand disease? "'Make sure that you avoid taking aspirin." "It's important that you avoid trauma. Clotting factor VIll can help your body compensate for the difficulty in clotting. Your disease affects your platelet function rather than clot formation

-Increase in the number of chair aerobics classes provided in the skilled care facilities -Interventions geared toward keeping the older adult population diagnosed with diabetes mellitus under tight blood glucose control by providing in-home cooking classes -Providing handwashing teaching sessions to a group of young children -Interests in keeping the older adult population engaged in such

While attending an international nursing conference, many discussions and break-out sessions focused on the World Health Organization's (WHO) views on health. Of the following comments made by nurses during a discussion session, which statements would be considered a good representation of the WHO definition? Select all that apply. -Increase in the number of chair aerobics classes provided in the skilled care facilities -Interventions geared toward keeping the older adult population diagnosed with diabetes mellitus under tight blood glucose control by providing in-home cooking classes -Providing handwashing teaching sessions to a group of young children -Interests in keeping the older adult population engaged in such activities as book reviews and word games during social time -Providing transportation for renal dialysis clients to and from their hemodialvsis sessions

Immediately discontinue the heparin therapy.

While being on subcutaneous heparin injections for deep vein thrombosis during the third trimester of her pregnancy, a client begins to experience major side effects. Her OB physician has called in a specialist who thinks the client is experiencing heparin-induced thrombocytopenia. The nurse should anticipate which order? Immediately discontinue the heparin therapy. A switch to warfarin 2.5 mg once/day. Infusion of FFP stat. Order to decrease the dose of heparin from 5000 units bid to 3000 units bid

During the procedure, an ultrasound will be utilized to guide the catheter into the correct position. Once the procedure is begun, you must lie very still since they will be inserting a needle through the uterine wall.

While preparing a client about to undergo percutaneous umbilical cord blood sampling (PUBS), which educational information should the nurse provide prior to the procedure? Select all that apply. O Pathophysiolo × O CH. 6: Genetic x + -We will put you into the stirrups and dilate your cervix with a small catheter so that we can obtain a cord sample. -We will send a sample of amniotic fluid to a regional medical center to have DNA tests performed for any genetic abnormality -During the procedure, an ultrasound will be utilized to guide the catheter into the correct position. -Once the procedure is begun, you must lie very still since they will be inserting a needle through the uterine wall.

If chromosome 21 is involved, there is a high risk for producing a child with Down syndrome.

While taking a daily walk, the nurse is asked by a neighbor what centric fusion (Robertsonian) translocation means. The neighbor tells the nurse that a family member has been diagnosed with this and is now afraid to have children. Given this diagnosis, what may be potential risks for her offspring? -If chromosome 21 is involved, there is a high risk for producing a child with Down syndrome. -This translocation of genetic material places the child at high risk for having multiple limb abnormalities. -Cleft lip with cleft palate is frequently associated with this translocation of genetic material. -Since the extremely short fragment only contains a small amount of genetic material, there should be no additional risk than the normal population.

Tetracycline for acne Isotretinoin for acne Warfarin for chronic atrial fibrillation Ethyl alcohol ingestion regularly several times per week

While taking a prenatal history, the nurse would be most concerned about severe teratogenic effects on the fetus if the mother admits to taking which medication prior to finding out that she was pregnant? Select all that apply. -Ethyl alcohol ingestion regularly several times per week -Tetracycline for ache -Over-the-counter cetirizine for seasonal allergies -Warfarin for chronic atrial fibrillation -Isotretinoin for acne

blindness and cataracts deafness

While traveling throughout Asia, a young couple was exposed to rubella. During their first clinic visit, the couple found out they were pregnant and express concern about their possible exposure to rubella. The nurse knows that this infant is at high risk for which possible complications related to rubella exposure? Select all that apply. Facial deformities like small palpebral fissures or thin vermillion border -Short, flipper-like appendages -Deafness - Small outbreak of blisters around the eyes and mouth 2 weeks after delivery back -Blindness or cataracts

A health care professional has recommended biofeedback to a client as a method of dealing with the high levels of stress. Which explanation should the nurse offer the client to explain biofeedback treatment?

You will be taught to gain control over skeletal muscle contractions

Discontinue the transfusion and begin an infusion of normal saline.

Your client with end-stage renal disease is receiving 2 units of packed red blood cells for anemia (Hgb of 8.2 g/dL [82 g/L)). Twenty minutes into the first transfusion, the nurse observes the client has a flushed face, hives over upper body trunk, and is reporting pain in lower back. His vital signs include pulse rate of 110 and BP drop to 95/56. What is the nurse's priority action? Discontinue the transfusion and begin an infusion of normal saline. Slow the rate of the blood infusion to 50 mL/hour. Recheck the type of blood infusing with the chart documentation of client's blood type. Document the assessment as the only action.

The nurse on the cardiac unit has noted that the client's potassium level is 6.1 mEq/L. The nurse has notified the physician and removed the banana from the client's meal tray. When explaining the nursing actions to the client, which statement is appropriate?

Your potassium level is high so I need you to let me know if you feel numbness, tingling, or weakness.

Hep B HPV

community health nurse is discussing preventative vaccines with a group of primigravda women. Which cancer-causing virus(es) should the nurse discuss that have a vaccine to prevent developing the disease? Select all that apply, ziKa virus meningitis hepatitis B chlamydia human papillomavirus (HPV)

During science class, a student asks, "What is the difference between plasma and serum in the blood?" The nurse responds that the primary difference between plasma and serum is that plasma contains: Heparin White blood cells Hydrogen ions Fibrinogen

fibrinogen

During a late night study session, a pathophysiology student reaches out to turn the page of her textbook. Which component of her nervous system contains the highest level of control of her arm and hand action? Thalamus Frontal lobe Basal ganglia Cerebellum

frontal lobe

A 41-year-old woman diagnosed with multiple sclerosis (MS) is sharing her story with members of an MS support group made up of people recently diagnosed. Which aspects of her health problem should the woman warn others to expect at some point in the progression of the disease? Select all that apply. Shuffling gait I Loss of mental acuity Gradual development of a resting tremor Debilitating fatigue Progressive loss of visual acuity

loss of mental acuity Debilitating fatigue Progressive loss of visual acuity

A client is asked to stand with feet together, eyes open, and hands by the sides. Then the client is asked to close the eyes while the clinician observes for a full minute. What assessment is the clinician performing? Posture Segmental reflex Proprioception Crossed-extensor reflex

proprioception

A clinician is conducting an assessment of a male client suspected of having a disorder of motor function. Which assessment finding would suggest a possible upper motor neuron (UMN) lesion? The client displays increased muscle tone. The client displays weakness in the distal portions of his limbs. The client has decreased deep tendon reflexes. The client's muscles appear atrophied.

the client displays increased muscle tone

The parents of a 3-year-old boy have brought him to a pediatrician for assessment of the boy's late ambulation and frequent falls. Subsequent muscle biopsy has confirmed a diagnosis of Duchenne muscular dystrophy. Which teaching point should the physician include when explaining the child's diagnosis to his parents? "Your son's muscular dystrophy is a result of faulty connections between muscles and the nerves that normally control them!' "He'll require intensive physical therapy as he grows up. and there's a good chance that he will outgrow this problem as he develops." "His muscles will weaken and will visibly decrease in size relative to his body size throughout his childhood" "Your son will be prone to heart problems and decreased lung function because of this."

your son will be prone to heart problems and decreased lung function because of this


Set pelajaran terkait

Rosh Review Pulmonology Fall 2017

View Set

Frontend Technical Interview Questions

View Set

AP Macro Final MC Questions: 1, 2, 3

View Set

Med Surg: Chapter 31: Nursing Management: Patients With Endocrine Disorders: PREPU

View Set